SlideShare a Scribd company logo
1 of 141
 In LP, the parameters (input data) of the model can
change within certain limits without causing the
optimum solution to change. This is referred to as
sensitivity analysis.
 Dual analysis deals with determining the new
optimum solution resulting from making targeted
changes in the input data.
3.1 – Sensitivity Analysis
2
 In this section two cases will be considered.
1. Sensitivity of the optimum solution to changes in the
availability of the resources (right-hand side of the
constraints).
2. Sensitivity of the optimum solution to changes in
unit profit or unit cost (coefficients of the objective
function).
3
3.1.1 – Graphical Sensitivity Analysis
Example 3.1 (Changes in the right hand side)
 JOBCO produces two products on two machines. A unit of
product 1 requires 2 hours on machine 1 and 1 hour on
machine 2. For product 2, a unit requires 1 hour on
machine 1 and 3 hours on machine 2. The revenues per
unit of products 1 and 2 are $30 and $20, respectively. The
total daily processing time available for each machine is 8
hours.
 Let X1 and X2 represent the daily number of units of
products 1 and 2, respectively.
4
3.1.1 – Graphical Sensitivity Analysis
Solution:
 The LP model is given as
Maximize z = 30X1 + 20X2
s.t.
2X1 + X2 ≤ 8 (Machine 1)
X1 + 3X2 ≤ 8 (Machine 2)
X1 , X2 ≥ 0 -----
 Figure 3.1 illustrates the change in the optimum
solution when changes are made in the capacity of
machine 1.
5
3.1.1 – Graphical Sensitivity Analysis
 If the daily
capacity is
increased from
8 hours to 9
hours, the new
optimum will
occur at point
G.
6
3.1.1 – Graphical Sensitivity Analysis
Figure 3.1
 The rate of change in optimum z resulting from changing
machine 1 capacity from 8 hours to 9 hours can be
computed as follows:
 The computed rate provides a direct link between the model
input (resources) and its output (total revenue). It says that a
unit increase (decrease) in machine 1 capacity will increase
(decrease) revenue by $14.
7
3.1.1 – Graphical Sensitivity Analysis
Rate of revenue change
resulting from incresing
machine 1 capacity by 1 hr
(point C to point G)
=
𝑧𝐺 − 𝑧𝐶
(capacity change)
=
142 − 128
9 − 8
= $14 /hr
 The name unit worth of a resource is an apt
description of the rate of change of the objective
function per unit change of a resource. Now it is
called dual (or shadow) price.
 In Fig. 3.1 it can be seen that the dual price of
$14.00/hr remains valid for changes (increases or
decreases) in machine 1 capacity that move its
constraint parallel to itself to any point on the line
segment BF. This means that the range of applicability
of the given dual price can be computed as follows:
8
3.1.1 – Graphical Sensitivity Analysis
Minimum machine 1 capacity [at B = (0, 2.67)]
= 2 × 0 + 1 × 2.67 = 2.67 hr
Maximum machine 1 capacity [at F = (8, 0)]
= 2 × 8 + 1 × 0 = 16 hr
 The conclusion is that the dual price of $14/hr will remain
valid for the range
2.67 hr ≤ Machine 1 capacity ≤ 16 hr
Changes outside this range will produce a different dual
price (worth per unit).
9
3.1.1 – Graphical Sensitivity Analysis
Minimum machine 1 capacity [at B = (0, 2.67)]
= 2 × 0 + 1 × 2.67 = 2.67 hr
Maximum machine 1 capacity [at F = (8, 0)]
= 2 × 8 + 1 × 0 = 16 hr
10
3.1.1 – Graphical Sensitivity Analysis
Capacity of M1
𝟐𝒙𝟏 + 𝒙𝟐
Z-value
𝟑𝟎𝒙𝟏 + 𝟐𝟎𝒙𝟐
𝐵 = (0, 2.67) 2 0 + 8/3 = 8/3 30 0 + 20 8/3 = 160/3
𝐹 = (8, 0) 2 8 + 0 = 16 30 8 + 20 0 = 240
Dual price =
∆ 𝑧_value
∆ capacity 𝑀1
=
240 − 160/3
16 − 8/3
= $14/ℎ𝑟
feasible range: 2.67 ≤ capacity of 𝑀1 ≤ 16
 By using similar computations, the dual price for machine 2
capacity is $2.00/hr and it remains valid for changes (increases
or decreases) that move its constraint parallel to itself to any
point on the line segment DE, which yields the following
limits:
11
3.1.1 – Graphical Sensitivity Analysis
Capacity of M2
𝒙𝟏 + 𝟑𝒙𝟐
Z-value
𝟑𝟎𝒙𝟏 + 𝟐𝟎𝒙𝟐
D = (4, 0) 4 + 3 0 = 4 30 4 + 20 0 = 120
E = (0, 8) 0 + 3 8 = 24 30 0 + 20 8 = 160
Dual price =
∆ 𝑧_value
∆ capacity 𝑀2
=
160 − 120
24 − 4
= $2/ℎ𝑟
feasibility range: 4 ≤ capacity of 𝑀2 ≤ 24
 The conclusion is that the dual price of $2.00/hr for
machine 2 will remain applicable for the range
4 hr ≤ Machine 2 capacity ≤ 24 hr
 The computed limits for machine 1 and 2 are referred
to as the feasibility ranges.
 The dual prices allow making economic decisions
about the LP problem, as the following questions
demonstrate:
12
3.1.1 – Graphical Sensitivity Analysis
 Question 1. If JOBCO can increase the capacity of both
machines, which machine should receive priority?
 Question 2. A suggestion is made to increase the
capacities of machines 1 and 2 at the additional cost of
$10/hr. Is this advisable?
13
3.1.1 – Graphical Sensitivity Analysis
 Question 3. If the capacity of machine 1 is increased
from the present 8 hours to 13 hours, how will this
increase impact the optimum revenue?
14
3.1.1 – Graphical Sensitivity Analysis
 Question 4. Suppose that the capacity of machine 1 is
increased to 20 hours, how will this increase impact the
optimum revenue?
 Remember that falling outside the feasibility range does
not mean that the problem has no solution. It only means
that available information is not sufficient to make a
complete decision.
15
3.1.1 – Graphical Sensitivity Analysis
 Question 5. The change in the optimum objective value
equals (dual price × change in resource) so long as the
change in the resource is within the feasibility range. How
can we determine the new optimum values of the variables
associated with a change in a resource?
16
3.1.1 – Graphical Sensitivity Analysis
Example 3.2 (Changes in the objective coefficient)
 Fig. 3.2 shows the graphical solution space of the
JOBCO problem in Example 3.1. The optimum occurs
at point C (X1 = 3.2, X2 = 1.6, Z = 128). Changes in
revenue units (i.e., objective-function coefficients)
will change the slope of z. However, as can be seen
from the figure, the optimum solution at point C
remains unchanged as long as the objective function
lies between lines BF and DE, the two constraints that
define the optimum point.
17
3.1.1 – Graphical Sensitivity Analysis
 This means that
there is a range
for the
coefficients of the
objective function
that will keep the
optimum solution
unchanged at C.
18
3.1.1 – Graphical Sensitivity Analysis
Figure 3.2
 To determine ranges for the coefficients of the
objective function that keep the optimum solution
unchanged at point C , first write the objective
function can in the general format
Maximize z = c1 X1 + c2 X2
Imagine now that the line z is pivoted at C and that it
can rotate clockwise and counterclockwise.
 The optimum solution will remain at point C so long
as z = c1 X1 + c2 X2 lies between the two lines X1 +
3X2 =8 and 2X1 + X2 = 8.
19
3.1.1 – Graphical Sensitivity Analysis
 This means that the ratio c1/c2 can vary between 1/3
and 2/1 , which yields the following optimality
range.
 This information can provide immediate answers
regarding the optimum solution as the following
questions demonstrate:
20
3.1.1 – Graphical Sensitivity Analysis
1
3
≤
𝑐1
𝑐2
≤
2
1
or 0.333 ≤
𝑐1
𝑐2
≤ 2
 Question 1. Suppose that the unit revenues for
products 1 and 2 are changed to $35 and $25,
respectively. Will the current optimum remain the
same?
21
3.1.1 – Graphical Sensitivity Analysis
 Question 2. Suppose that the unit revenue of product
2 is fixed at its current value c2 = $20. What is the
associated optimality range for the unit revenue for
product 1, c1 , that will keep the optimum unchanged?
22
3.1.1 – Graphical Sensitivity Analysis
Exercise 3.1:
 Wild West produces two types of cowboy hats. A Type 1
hat requires twice as much labor time as a Type 2. If all
the available labor time is dedicated to Type 2 alone, the
company can produce a total of 400 Type 2 hats a day. The
respective market limits for the two types are 150 and 200
hats per day. The revenue is $8 per Type 1 hat and $5 per
Type 2 hat.
a) Use the graphical solution to determine the number of
hats of each type that maximizes revenue.
23
3.1.1 – Graphical Sensitivity Analysis
b) Determine the dual price of the production capacity (in terms
of the Type 2 hat) and the range for which it is applicable.
c) If the daily demand limit on the Type 1 hat is decreased to
120, use the dual price to determine the corresponding effect
on the optimal revenue.
d) What is the dual price of the market share of the Type 2 hat?
By how much can the market share be increased while
yielding the computed worth per unit?
e) Determine the optimality range for the unit revenue ratio of
the two types of hats that will keep the current optimum
unchanged.
f) Using the information in (e), will the optimal solution change
if the revenue per unit is the same for both types?
24
3.1.1 – Graphical Sensitivity Analysis
Solution:
25
3.1.1 – Graphical Sensitivity Analysis
26
3.1.1 – Graphical Sensitivity Analysis
 This section extends the sensitivity analysis to
the general LP model to determine the dual
price by using algebraic calculation.
 A numeric example (the TOYCO model) will
be used to facilitate the presentation.
27
3.1.2 – Algebraic Sensitivity Analysis
Example 3.3 (TOYCO Model)
 TOYCO uses three operations to assemble three types of
toys-trains, trucks, and cars. The daily available times for
the three operations are 430, 460, and 420 minutes,
respectively, and the revenues per unit of toy train, truck,
and car are $3, $2, and $5, respectively. The assembly
times per train at the three operations are 1, 3, and 1
minutes, respectively. The corresponding times per train
and per car are (2,0,4) and (1,2,0) minutes (a zero time
indicates that the operation is not used).
28
3.1.2 – Algebraic Sensitivity Analysis
Solution:
 Letting X1 , X2 , and X3 represent the daily number of units
assembled of trains, trucks, and cars, respectively, the
associated LP model is given as:
Maximize z = 3X1 + 2X2 + 5X3
s.t.
X1 + 2X2 + X3 ≤ 430 (operation 1)
3X1 + 2X3 ≤ 460 (operation 2)
X1 + 4X2 ≤ 420 (operation 3)
X1 , X2 , X3 ≥ 0 (operation 1)
 Using X4 , X5 , and X6 as the slack variables
29
3.1.2 – Algebraic Sensitivity Analysis
 The optimal tableau is
 The solution recommends manufacturing 100 trucks and
230 cars but no trains. The associated revenue is $1350.
30
3.1.2 – Algebraic Sensitivity Analysis
Basic X1 X2 X3 X4 X5 X6 Solution
z 4 0 0 1 2 0 1350
X2 -1/4 1 0 ½ -1/4 0 100
X3 3/2 0 1 0 ½ 0 230
X6 2 0 0 -2 1 1 20
Determination of Dual Prices and feasibility
ranges
 Recognizing that the dual prices and their feasibility
ranges are rooted in making changes in the right-hand
side of the constraints, suppose that D1 , D2 , and D3
are the (positive or negative) changes made in the
allotted daily manufacturing time of operations 1, 2,
and 3, respectively.
 The original TOYCO model can then be changed to
31
3.1.2 – Algebraic Sensitivity Analysis
Maximize z = 3X1 + 2X2 + 5X3
s.t.
X1 + 2X2 + X3 ≤ 430 + D1 (operation 1)
3X1 + 2X3 ≤ 460 + D2 (operation 2)
X1 + 4X2 ≤ 420 + D3 (operation 3)
X1 , X2 , X3 ≥ 0 (operation 1)
 First rewrite the starting tableau using the new right-hand
side, 430 + D1 , 460 + D2 , and 420 + D3 .
32
3.1.2 – Algebraic Sensitivity Analysis
 The two shaded areas are identical. Hence, if we repeat the
same simplex iterations as in the original model, the
columns in the two groups will also be identical in the
optimal tableau.
33
3.1.2 – Algebraic Sensitivity Analysis
Solution
Basic X1 X2 X3 X4 X5 X6 RHS D1 D2 D3
z -3 -2 -5 0 0 0 0 0 0 0
X4 1 2 1 1 0 0 430 1 0 0
X5 3 0 2 0 1 0 460 0 1 0
X6 1 4 0 0 0 1 420 0 0 1
 That is
34
3.1.2 – Algebraic Sensitivity Analysis
Solution
Basic X1 X2 X3 X4 X5 X6 RHS D1 D2 D3
z 4 0 0 1 2 0 1350 1 2 0
X2 -1/4 1 0 ½ -1/4 0 100 ½ -1/4 0
X3 3/2 0 1 0 ½ 0 230 0 ½ 0
X6 2 0 0 -2 1 1 20 -2 1 1
 The new optimum tableau provides the following
optimal solution:
z = 1350 + D1 + 2D2
X2 = 100 + 1/2D1 – 1/4D2
X3 = 230 + 1/2D2
X6 = 20 - 2D1 + D2 + D3
 Now use this solution to determine the dual prices and
the feasibility ranges.
35
3.1.2 – Algebraic Sensitivity Analysis
Dual prices:
 The value of the objective function can be written as
z = 1350 + 1D1 + 2D2 + 0D3
 This means that, by definition, the corresponding dual
prices are 1, 2, and 0 ($/min) for operations 1, 2, and 3,
respectively.
 The coefficients of D1 , D2 , and D3 in the optimal z-row
are exactly those of the slack variables X4 , X5 , and X6.
This means that the dual prices equal the coefficients of
the slack variables in the optimal z-row.
36
3.1.2 – Algebraic Sensitivity Analysis
Feasibility range:
 The current solution remains feasible if all the basic
variables remain nonnegative:
X2 = 100 + 1/2D1 – 1/4D2 ≥ 0
X3 = 230 + 1/2D2 ≥ 0
X6 = 20 - 2D1 + D2 + D3 ≥ 0
 Simultaneous changes D1 , D2 , and D3 that satisfy
these inequalities will keep the solution feasible
37
3.1.2 – Algebraic Sensitivity Analysis
 Suppose that the manufacturing time available for
operations 1, 2, and 3 are 480, 440, and 400 minutes
respectively. Then
D1 =480–430= 50, D2=440 –460=-20, D3=400–420= -20
Substituting in the feasibility conditions
X2 = 100 + ½(50)– ¼(-20)= 130 > 0  Feasible
X3 = 230 + ½(-20)= 220 > 0  Feasible
X6 = 20 – 2(50) + (-20) + (-20)= -120 < 0  Infeasible
38
3.1.2 – Algebraic Sensitivity Analysis
 If D1 = -30, D2= –12, D3= 10 , then
X2 = 100 + ½(-30)– ¼(-12)= 88 > 0  Feasible
X3 = 230 + ½(-12)= 224 > 0  Feasible
X6 = 20 – 2(-30) + (-12) + (10)= 78 < 0  Feasible
The new optimal solution is X2 = 88, X3 = 224, X6 = 78
with
z = 1350 + 1(-30) + 2(-12) + 0(10) = $1296.
 The given conditions can produce the individual feasibility
ranges associated with changing the resources one at a
time.
39
3.1.2 – Algebraic Sensitivity Analysis
 For example, a change of time only for operation 1 means
that D2 = D3 = 0. So,
X2 = 100 + 1/2D1 ≥ 0
X3 = 230 ≥ 0  -200 ≤ D1 ≤ 10
X6 = 20 - 2D1 ≥ 0
Similarly, -20 ≤ D2 ≤ 400 and -20 ≤ D3 < ∞
40
3.1.2 – Algebraic Sensitivity Analysis
Resource amount (minutes)
Resource Dual price ($) Feasibility range Minimum Current Maximum
Operation 1 1 -200 ≤ D1 ≤ 10 230 430 440
Operation 2 2 -20 ≤ D2 ≤ 400 440 460 860
Operation 3 0 -20 ≤ D3 < ∞ 400 420 ∞
Exercise 3.2:
 For problem in exercise 1 find dual price and
feasibility ranges.
Maximize z = 8X1 + 5X2
s.t.
2X1 + X2 ≤ 400 + D1
X1 ≤ 150 + D2
X2 ≤ 200 + D3
X1 , X2 ≥ 0
41
3.1.2 – Algebraic Sensitivity Analysis
Solution:
42
3.1.2 – Algebraic Sensitivity Analysis
Determination of the optimality ranges -
Changes in objective coefficients
 In the TOYCO model, let d1 , d2 , and d3 represent the
change in unit revenues for toy trucks, trains, and cars,
respectively.
Maximize z = 3X1 + 2X2 + 5X3
s.t.
X1 + 2X2 + X3 ≤ 430 (operation 1)
3X1 + 2X3 ≤ 460 (operation 2)
X1 + 4X2 ≤ 420 (operation 3)
X1 , X2 , X3 ≥ 0 (operation 1)
43
3.1.2 – Algebraic Sensitivity Analysis
 The objective function then becomes
Maximize z = (3+ d1)X1 + (2+ d2)X2 + (5+ d3)X3
 With the simultaneous changes, the z-row in the starting
tableau appears as:
44
3.1.2 – Algebraic Sensitivity Analysis
Basic X1 X2 X3 X4 X5 X6 Solution
z -3- d1 -2- d2 -5- d3 0 0 0 0
X4 1 2 1 1 0 0 430
X5 3 0 2 0 1 0 460
X6 1 4 0 0 0 1 420
 When we generate the simplex tableaus using the
same sequence of entering and leaving variables in the
original model (before the changes dj are introduced),
the optimal iteration will appear as follows
45
3.1.2 – Algebraic Sensitivity Analysis
Basic X1 X2 X3 X4 X5 X6 Solution
z 4-1/4d2 +3/2d3-d1 0 0 1+1/2d2 2-1/4d2 +1/2d3 0
1350+100d2
+230d3
X2 -1/4 1 0 ½ -1/4 0 100
X3 3/2 0 1 0 ½ 0 230
X6 2 0 0 -2 1 1 20
 The new optimal tableau is the same as in the original
optimal tableau except for the z-equation coefficients. This
means that changes in the objective-function coefficients
can affect the optimality of the problem only.
( Changes in the right-hand side affect feasibility only)
 An examination of the new z-row shows that the
coefficients of dj are taken directly from the constraint
coefficients of the optimum tableau. A convenient way for
computing is to add a new top row and a new leftmost
column to the optimum tableau, as shown by the shaded
areas below.
46
3.1.2 – Algebraic Sensitivity Analysis
 The entries in the top row are the change dj associated
with variable Xj. For the leftmost column, the top element
is 1 in the z-row followed by change dj for basic variable
Xj. Keep in mind that dj = 0 for the slack variables.
z = (3+ d1)X1 + (2+ d2)X2 + (5+ d3)X3
47
3.1.2 – Algebraic Sensitivity Analysis
d1 d2 d3 0 0 0
Basic X1 X2 X3 X4 X5 X6 Solution
1 z 4 0 0 1 2 0 1350
d2 X2 -1/4 1 0 ½ -1/4 0 100
d3 X3 3/2 0 1 0 ½ 0 230
0 X6 2 0 0 -2 1 1 20
 To compute the new objective coefficients of nonbasic
variables (or the value of z), multiply the elements of
its column by the corresponding elements in the
leftmost column, add them up, and subtract the top-
row element from the sum. For example, for X1 , we
have
Objective coefficient of X1
= [4 × 1 + (-1/4) × d2 + (3/2) × d3 + 2 × 0] – d1
= 4 – 1/4 d2 + 3/2 d3 - d1
48
3.1.2 – Algebraic Sensitivity Analysis
 The current solution remains optimal so long as the new z-
equation coefficients remain nonnegative (maximization
case). We thus have the following simultaneous
optimality conditions corresponding to nonbasic X1 , X4 ,
and X5 :
4 – 1/4 d2 + 3/2 d3 - d1 ≥ 0
1 + 1/2 d2 ≥ 0
2 – 1/4 d2 + 1/2 d3 ≥ 0
 Remember that the z-equation coefficient for a basic
variable is always zero.
49
3.1.2 – Algebraic Sensitivity Analysis
 To illustrate the use of these conditions, suppose that the
objective function of TOYCO is changed from
z = 3X1 + 2X2 + 5X3 to z = 2X1 + X2 + 6X3 . Then,
d1 = 2 – 3 = -$1 , d2 = 1 – 2 = -$1 and d3 = 6 – 5 = $1
substitution in the given conditions yields
4 – 1/4 d2 + 3/2 d3 - d1 = 4 –1/4 (-1)+3/2 (1)- (-1)= 6.75 >
0
1 + 1/2 d2 = 1 + ½ (-1) = 0.5 > 0
2 – 1/4 d2 + 1/2 d3 = 2 – ¼ (-1) + ½ (1) = 2.75 > 0
all are satisfied.
50
3.1.2 – Algebraic Sensitivity Analysis
 The results show that the proposed changes will keep the
current solution (X1 = 0, X2 = 100, X3 = 230) optimal
(with a new value of
z = 1350 + 100d2 + 230d3 = 1350 + 100(-1)+ 230(1)
= $1480
If any of the conditions is not satisfied, a new solution
must be determined.
 The only difference in the minimization case is that the z-
equations coefficients must be ≤ 0 to maintain optimality.
51
3.1.2 – Algebraic Sensitivity Analysis
 The optimality ranges dealing with changing dj one at a
time can be developed from the simultaneous optimality
conditions. For example, suppose that the objective
coefficient of X2 only is changed to 2 + d2 meaning that d1
= d3 = 0. the simultaneous optimality conditions thus
reduce to
4 – 1/4 d2 ≥ 0  d2 ≤ 16
1 + 1/2 d2 ≥ 0  d2 ≥ -2  -2 ≤ d2 ≤ 8
2 – 1/4 d2 ≥ 0  d2 ≤ 8
Also, d1 < 4 and d3 ≥ -8/3
52
3.1.2 – Algebraic Sensitivity Analysis
 The given individual conditions can be translated to
total unit revenue ranges. For example, for toy trucks
(variable X2), the total unit revenue is 2 + d2 and its
optimality range -2 ≤ d2 ≤ 8 translates to
2 -2 ≤ 2+d2 ≤ 2+8
$0 ≤ c2 (unit revenue of toy truck) ≤ $10
 It assumes that the unit revenues for toy trains and toy
cars remain fixed at $3 and $5, respectively.
53
3.1.2 – Algebraic Sensitivity Analysis
 It is important to notice that the changes d1 , d2 , and d3
may be within their allowable individual ranges without
satisfying the simultaneous conditions, and vice versa. For
example, consider z = 6X1 + 8X2 + 3X3 .
Here, d1 = 6-3= 3, d2 = 8-2= 6, and d3 = 3-5= -2
(-∞ < d1 < 4 , -2 ≤ d2 ≤ 8 , and -8/3 ≤ d3 < ∞)
However, the corresponding simultaneous conditions yield
4 – 1/4 d2 + 3/2 d3 - d1 = 4 –1/4 (6)+3/2 (-2)- 3= -3.5 < 0
1 + 1/2 d2 = 1 + ½ (6) = 4 > 0
2 – 1/4 d2 + 1/2 d3 = 2 – ¼ (6) + ½ (-2) = -0.5 < 0
54
3.1.2 – Algebraic Sensitivity Analysis
Exercise 3.3:
 HiDec produces two models of electronic gadgets that use
resistors, capacitors, and chips. The following table
summarizes the data of the situation:
55
3.1.2 – Algebraic Sensitivity Analysis
Unit resource requirements
Resource
Model 1
(units)
Model 2
(units)
Maximum availability
(units)
Resistor 2 3 1200
Capacitor 2 1 1000
Chips 0 4 800
Unit price ($) 3 4
a) Determine the status of each resource.
b) In terms of the optimal revenue, determine the dual
prices for the resistors, capacitors, and chips.
c) Determine the feasibility ranges for the dual prices
obtained in (b).
d) If the available number of resistors is increased to
1300 units, find the new optimum solution.
e) If the available number of chips is reduced to 350
units, will you be able to determine the new
optimum solution directly from the given
information? Explain.
56
3.1.2 – Algebraic Sensitivity Analysis
f) If the availability of capacitors is limited by the
feasibility range computed in (c), determine the
corresponding range of the optimal revenue and the
corresponding ranges for the numbers of units to be
produced of Models 1 and 2.
g) A new contractor is offering to sell HiDec additional
resistors at 40 cents each, but only if HiDec would
purchase at least 500 units. Should HiDec accept the
offer?
57
3.1.2 – Algebraic Sensitivity Analysis
Solution:
58
3.1.2 – Algebraic Sensitivity Analysis
59
3.1.2 – Algebraic Sensitivity Analysis
Exercise 3.4:
 The Bank of Elkins is allocating a maximum of $200,000 for
personal and car loans during the next month. The bank
charges 14% for personal loans and 12% for car loans. Both
types of loans are repaid at the end of a 1-year period.
Experience shows that about 3% of personal loans and 2% of
car loans are not repaid. The bank usually allocates at least
twice as much to car loans as to personal loans.
a) Determine the optimal allocation of funds between the two
loans and the net rate of return on all the loans.
b) If the percentages of personal and car loans are changed to
4% and 3%, respectively, use sensitivity analysis to
determine jf the optimum solution in (a) will change.
60
3.1.2 – Algebraic Sensitivity Analysis
Solution:
61
3.1.2 – Algebraic Sensitivity Analysis
62
3.1.2 – Algebraic Sensitivity Analysis
 The dual problem is defined systematically from the
primal (or original) LP model. The two problems are
closely related, in the sense that the optimal solution of
one problem automatically provides the optimal solution
to the other.
 In most LP treatments, the dual is defined for various
forms of the primal depending on the sense of
optimization (maximization or minimization), types of
constraints (≤, ≥, or =), and sign of the variables
(nonnegative or unrestricted).
63
3.2 – Duality
 The present definition of the dual problem requires
expressing the primal problem in the equation form
(all the constraints are equations with nonnegative
right-hand side and all the variables are nonnegative).
 This requirement is consistent with the format of the
simplex starting tableau. Hence, any results obtained
from the primal optimal solution will apply directly to
the associated dual problem.
64
3.2 – Duality
 Steps for constructing the dual from the primal are :
1) Assign a dual variable for each primal (equality)
constraint.
2) Construct a dual constraint for each primal variable.
3) The column (constraint) coefficients and the objective
coefficient of the jth primal variable respectively define
the left-hand and the right-hand sides of the jth dual
constraint.
4) The dual objective coefficients equal the right-hand
sides of the primal constraint equations.
65
3.2 – Duality
 The sense of optimization, direction of inequalities,
and the signs of the variables in the dual are in
following table.
66
3.2 – Duality
Dual problem
Primal problem
objective
Objective Constrains type Variable sign
Maximization Minimization ≥ Unrestricted
Minimization Maximization ≤ Unrestricted
Example 3.4
 Primal model:
Maximize z = 5X1 + 12X2 + 4X3
S.t.
X1 + 2X2 + X3 ≤ 10
2X1 – X2 + 3X3 = 8
X1 , X2 , X3 ≥ 0
 Equation form:
Max z = 5X1 + 12X2 + 4X3 + 0X4
S.t. Dual variables
X1 + 2X2 + X3 + X4 = 10  y1
2X1 – X2 + 3X3 + 0X4 = 8  y2
X1 , X2 , X3 , X4 ≥ 0
67
3.2 – Duality
 Dual problem
Minimize w = 10y1 + 8y2
S.t.
y1 + 2y2 ≥ 5
2y1 - y2 ≥ 12
y1 + 3y2 ≥ 4
y1 + 0y2 ≥ 0
y1 , y2 unrestricted  y1 ≥ 0 ,
y2 unrestricted
68
3.2 – Duality
Example 3.5
 Primal model:
Minimize z = 15X1 + 12X2
S.t.
X1 + 2X2 ≥ 3
2X1 – 4X2 ≤ 5
X1 , X2 ≥ 0
 Equation form:
69
3.2 – Duality
 Dual problem:
70
3.2 – Duality
Example 3.6
 Primal model:
Maximize z = 5X1 + 6X2
S.t.
X1 + 2X2 = 5
-X1 + 5X2 ≥ 3
4X1 + 7X2 ≤ 8
X1 unrestricted, X2 ≥ 0
71
3.2 – Duality
 Equation form:
72
3.2 – Duality
 The first and second constraints are replaced by an
equation. The general rule in this case is that an
unrestricted primal variable always corresponds to an
equality dual constraint. Conversely, a primal equation
produces an unrestricted dual variable, as the first
primal constraint demonstrates.
 Note: No provision is made for including artificial
variables in the primal, because artificial variables
would not change the definition of the dual problem.
73
3.2 – Duality
 The following table summarizes the primal-dual rules:
74
3.2 – Duality
Maximization problem Minimization problem
Constraints Variables
≥  ≤ 0
≤  ≥ 0
=  Unrestricted
Variables Constraints
≥ 0  ≥
≤ 0  ≤
Unrestricted  =
3.2.1 - Primal-Dual Relationships
 Changes made in the original LP model can affect the
optimality and/or the feasibility of the current
solution. This section introduces a number of primal-
dual relationships that can be used to recompute the
elements of the optimal simplex tableau.
75
3.2 – Duality
Starting Variables
Objective
z-row
=
Constraint
columns
1 0 … 0
0 1 … 0
⁞ ⁞ … 0 =
0 0 … 1
Identity matrix
(Starting Tableau)
76
2.2 – The Simplex Method
Figure 3.4 (a)
77
2.2 – The Simplex Method
Starting Variables
Objective
z-row
=
Constraint
columns
Inverse matrix
=
(General Iteration)
Figure 3.4 (b)
 Figure 3.4 gives a schematic representation of the
starting and general simplex tableaus. In the starting
tableau, the constraint coefficients under the starting
variables form an identity matrix (all main-diagonal
elements equal 1 and all off-diagonal elements equal
zero). With this arrangement, subsequent iterations of
the simplex tableau generated by the Gauss-Jordan
row operations will modify the elements of the
identity matrix to produce what is known as the
inverse matrix. The inverse matrix is key to
computing all the elements of the associated simplex
tableau.
78
3.2.2 – Simplex Tableau Layout
3.2.3 – Optimal Dual Solution
 The primal and dual solutions are closely related that
the optimal solution of either problem directly yields
the optimal solution to the other. Thus, in an LP model
in which the number of variables is considerably
smaller than the number of constraints, computational
savings may be realized by solving the dual because
the amount of simplex computation depends largely
(though not totally) on the number of constraints.
79
3.2 – Duality
This section provides two methods for determining the
dual values.
 Method 1.
 Method 2.
80
3.2.3 – Optimal Dual Solution
Optimal values
of 𝑑𝑢𝑎𝑙 variables
=
Row vector of
original objective coefficients
of optimal 𝑝𝑟𝑖𝑚𝑎𝑙 basic variables
×
Optimal 𝑝𝑟𝑖𝑚𝑎𝑙
inverse
Optimal value of
dual variable yi
=
Optimal primal z-coefficient of starting variable xi
+
original objective coefficient of xi
 The elements of the row vector must appear in the
same order the basic variables are listed in the Basic
column of the simplex tableau.
81
3.2.3 – Optimal Dual Solution
Example 3.7
 Consider the following LP:
Maximize z = 5X1 + 12X2 + 4X3
S.t.
X1 + 2X2 + X3 ≤ 10
2X1 – X2 + 3X3 = 8
X1 , X2 , X3 ≥ 0
 Primal:
Max z = 5X1 + 12X2 + 4X3 - MR
S.t. Dual variables
X1 + 2X2 + X3 + X4 = 10  y1
2X1 – X2 + 3X3 + R = 8  y2
X1 , X2 , X3 , X4 , R ≥ 0
82
3.2.3 – Optimal Dual Solution
 Dual problem
Minimize w = 10y1 + 8y2
S.t.
y1 + 2y2 ≥ 5
2y1 - y2 ≥ 12
y1 + 3y2 ≥ 4
y1 ≥ 0
y2 ≥ -M  y2 unrestricted
83
3.2.3 – Optimal Dual Solution
 Optimal tableau of the primal
 In this table the starting primal variables X4 and R
uniquely correspond to the dual variables yl and y2 ,
respectively. Thus, we determine the optimum dual
solution as follows:
84
3.2.3 – Optimal Dual Solution
Basic X1 X2 X3 X4 R Solution
z 0 0 3/5 29/5 -2/5+ M 274/5
X2 0 1 -1/5 2/5 -1/5 12/5
X1 1 0 7/5 1/5 2/5 26/5
 Method 1.
85
3.2.3 – Optimal Dual Solution
Starting primal basic variables X4 R
Z-equation coefficients 29/5 -2/5+ M
Original objective coefficient 0 -M
Dual variables y1 y2
Optimal dual values 29/5 + 0 = 29/5 -2/5 +M+(-M)= -2/5
 Method 2.
The optimal inverse matrix, highlighted in the table under
the starting variables X4 and R, is
The order of the optimal primal basic variables in the
Basic column is X2 followed by X1. The elements of the
original objective coefficients for the two variables must
appear in the same order-namely,
86
3.2.3 – Optimal Dual Solution
optimal inverse =
2/5 −1/5
1/5 2/5
(Original objective coefficients)=coefficient of (X2 , X1)
= (12, 5)
 The optimal values are
87
3.2.3 – Optimal Dual Solution
(𝑦1 𝑦2) = (12 5) ×
2
5
−
1
5
1
5
2
5
= (29/5 −2/5)
Primal-Dual objective values:
 For any pair of feasible primal and dual solutions,
 At the optimum, the relationship holds as a strict equation. The
relationship does not specify which problem is primal and
which is dual. Only the sense of optimization (maximization or
minimization) is important in this case.
88
3.2.3 – Optimal Dual Solution
Objective value in the
𝑚𝑎𝑥𝑖𝑚𝑖𝑧𝑎𝑡𝑖𝑜𝑛 problem
≤
Objective value in the
𝑚𝑖𝑛𝑖𝑚𝑖𝑧𝑎𝑡𝑖𝑜𝑛 problem
Figure 3.5
Example 3.8
 In Example 3.7, (X1 = 0, X2 = 0, X3 = 8/3) and ( y1 =
6, y2 = 0) are (arbitrary) feasible primal and dual
solutions. The associated values of the objective
functions are
z = 5X1 + 12X2 + 4X3 = 5(0) + 12(0) + 4(8/3) = 32/3
w = 10y1 + 8y2 = 10(6) + 8(0) = 60
Thus, z (=32/3) for the maximization problem
(primal) is less than w (=60) for the minimization
problem (dual). The optimum value of z (= 274/5)
falls within the range (32/3, 60).
89
3.2.3 – Optimal Dual Solution
Exercise 3.5:
 Estimate a range for the optimal objective value for the
following LP:
Minimize z = 5X1 + 2X2
S.t.
X1 - X2 ≥ 3
2X1 + 3X2 ≥ 5
X1 , X2 ≥ 0
90
3.2.3 – Optimal Dual Solution
Solution:
 Dual problem:
91
3.2.3 – Optimal Dual Solution
3.2.4 - Simplex Tableau Computations
 This section shows how any iteration of the entire
simplex tableau can be generated from the original
data of the problem, the inverse associated with the
iteration, and the dual problem. Using the layout of
the simplex tableau in Figure 3.4, we can divide the
computations into two types:
1) Constraint columns (left- and right-hand sides).
2) Objective z-row.
92
3.2 – Duality
93
Figure 3.4
3.2.4 – Simplex Tableau Computations
 Formula 1: Constraint Column Computations.
In any simplex iteration, a left-hand or a right-hand side
column is computed as follows:
 Formula 2: Objective z-row Computations.
In any simplex iteration, the objective equation coefficient
of xi is computed as follows:
94
3.2.4 – Simplex Tableau Computations
Constraint column
in iteration 𝑖
=
Inverse in
iteration 𝑖
×
Original
constraint column
Primal z-equation
coefficient of variable xi
=
Left-hand side of
jth dual constraint
–
Right-hand side of
jth dual constraint
.
Example 3.9
 Consider the LP in Example 3.7, use the formulas 1 and 2
and find the coefficient of X1 in optimum tableau.
Maximize z = 5X1 + 12X2 + 4X3
S.t.
X1 + 2X2 + X3 ≤ 10
2X1 – X2 + 3X3 = 8
X1 , X2 , X3 ≥ 0
95
3.2.4 – Simplex Tableau Computations
Solution:
 From the table
96
3.2.4 – Simplex Tableau Computations
Basic X1 X2 X3 X4 R Solution
z 0 0 3/5 29/5 -2/5+ M 274/5
X2 0 1 -1/5 2/5 -1/5 12/5
X1 1 0 7/5 1/5 2/5 26/5
optimal inverse =
2/5 −1/5
1/5 2/5
 Formula 1:
Similar computation can be used to generate the
optimal columns for X2, X3, X4, and R.
97
3.2.4 – Simplex Tableau Computations
𝑥1column in
optimal iteration
=
Inverse in
optimal iteration
×
Original
x1column
=
2/5 −1/5
1/5 2/5
×
1
2
=
0
1
 Formula 2:
The optimal values of the dual variables, (y1, y2)= (29/5, -
2/5), are used in formula 2 to compute all the z-
coefficients.
z-coefficient of X1 = LHS of constraint 1 – RHS
z-coefficient of X1 = y1 + 2y2 – 5 = 29/5+ 2(-2/5)- 5 = 0
z-coefficient of R = y2 – (-M) = -2/5 – (-M) = -2/5 +M
Similar computation can be used to determine the
z-coefficients for X2, X3, and X4 .
98
3.2.4 – Simplex Tableau Computations
Exercise 3.6
 Consider the following LP:
Minimize z = 2X1 + X2
S.t.
3X1 + X2 - X3 = 3
4X1 + 3X2 – X4 = 6
X1 + 2X2 + X5 = 3
X1 , X2 , X3 , X4 , X5 ≥ 0
99
3.2.4 – Simplex Tableau Computations
 Compute the entire simplex tableau associated with
the following basic solution and check it for
optimality and feasibility.
Basic variables = (X1 , X2 , X5 ),
Inverse =
100
3.2.4 – Simplex Tableau Computations
3
5 − 1
5 0
− 4
5
3
5 0
1 −1 1
Solution:
101
3.2.4 – Simplex Tableau Computations
 This section presents two additional algorithms: The
dual simplex and the generalized simplex. The dual
simplex starts infeasible (but better than optimal) and
remains infeasible until feasibility is restored at the
last iteration.
 The generalized simplex combines both the primal
and dual simplex methods, starting both nonoptimal
and infeasible. At the final iteration, the solution
becomes optimal and feasible (assuming that one
exists).
102
3.3 – Additional Simplex Algorithm
 When we use the simplex method to solve a max problem,
we begin with an initial solution that is primal feasible
(nonnegative rhs) but dual infeasible (at least one variable
in the z-row has a negative coefficient). Eventually, we
will obtain an optimal solution that is both primal feasible
and dual feasible (nonnegative z-row).
 When we use the dual simplex method to solve a max
problem, we begin with an initial solution that is dual
feasible (nonnegative z-row) but primal infeasible (at least
one constraint has a negative rhs). Eventually, we will
obtain an optimal solution that is both dual feasible and
primal feasible (nonnegative rhs).
103
3.3.1 – Dual Simplex Algorithm
 Dual feasibility condition. The leaving variable, xr is
the basic variable having the most negative value (ties are
broken arbitrarily). If all the basic variables are
nonnegative, the algorithm ends.
 Dual optimality condition. Given that xr is the leaving
variable, let be the objective coefficient of nonbasic
variable xj and αrj the constraint coefficient in the xr-row
and xj-column of the tableau. The entering variable is the
nonbasic variable with αrj < 0 that corresponds to
104
3.3.1 – Dual Simplex Algorithm
𝑐𝑗
min
Nonbasic 𝑥𝑗
𝑐𝑗
𝛼𝑟𝑗
, 𝛼𝑟𝑗 < 0
(Ties are broken arbitrarily.) If αrj ≥ 0 for all nonbasic xj ,
the problem has no feasible solution.
 To start the LP optimal and infeasible, two requirements
must be met:
1. The objective function must satisfy the optimality
condition of the regular simplex method.
2. All the constraints must be of the type (≤).
Inequalities of the type (≥) are converted to (≤) by
multiplying both sides of the inequality by -1. If the LP
includes (=) constraints, the equation can be replaced by
two inequalities.
105
3.3.1 – Dual Simplex Algorithm
Example 3.10
Minimize z = 3x1 + 2x2 + x3
subject to
3x1 + 2x2 + x3 ≥ 3
-3x1 + 3x2 + x3 ≥ 6
x1 + x2 + x3 ≤ 3
x1 , x2 , x3 ≥ 0
In the present example, the first two inequalities are
multiplied by -1 to convert them to (≤) constraints.
106
3.3.1 – Dual Simplex Algorithm
 The starting tableau is thus given as:
 The tableau is optimal because all the objective
coefficients in the z-row are nonnegative. It is also
infeasible because at least one of the basic variables is
negative (x4 = -3, x5 = -6, x6 = 3).
107
3.3.1 – Dual Simplex Algorithm
Basic x1 x2 x3 x4 x5 x6 Solution
z -3 -2 -1 0 0 0 0
x4 -3 -2 -1 1 0 0 -3
x5 3 -3 -1 0 1 0 -6
x6 1 1 1 0 0 1 3
 According to the dual feasibility condition, x5 (= -6) is
the leaving variable. The next table shows how the
dual optimality condition is used to determine the
entering variable.
 The ratios show that x2 is the entering variable.
108
3.3.1 – Dual Simplex Algorithm
j = 1 j = 2 j = 3
Nonbasic variable x1 x2 x3
z-row -3 -2 -1
x5-row, α5j 3 -3 -1
Ratio, | / α5j |, α5j <0 - 2/3 1
𝑐𝑗
𝑐𝑗
 The next tableau is obtained by using the familiar row
operations, which gives:
 Which is both optimal and feasible.
109
3.3.1 – Dual Simplex Algorithm
Basic x1 x2 x3 x4 x5 x6 Solution
z -5 0 -1/3 0 -2/3 0 4
x4 -5 0 -1/3 1 -2/3 0 1
x2 -1 1 1/3 0 -1/3 0 2
x6 2 0 2/3 0 1/3 1 1
 Notice how the dual simplex works. In all the
iterations, optimality is maintained (all coefficients are
≤o).At the same time, each new iteration moves the
solution toward feasibility.
110
3.3.1 – Dual Simplex Algorithm
Exercise 3.7
 Solve the following problem by using the dual simplex
algorithm.
Minimize z = 4x1 + 2x2
subject to
x1 + x2 = 10
3x1 - x2 ≥ 20
x1 , x2 ≥ 0
111
3.3.1 – Dual Simplex Algorithm
Solution:
112
3.3.1 – Dual Simplex Algorithm
 The (primal) simplex algorithm starts feasible but
nonoptimal. The dual simplex starts better than
optimal but infeasible.
What if an LP model starts both nonoptimal and
infeasible? Of course we can use artificial variables
and artificial constraints to secure a starting solution.
This is not mandatory, however, because the key idea
of both the primal and dual simplex methods is that
the optimum feasible solution, when finite, always
occurs at a corner point (or a basic solution).
113
3.3.2 – Generalized Simplex Algorithm
 This suggests that a new simplex algorithm can be
developed based on tandem use of the dual simplex
and primal simplex methods. First, use the dual
algorithm to get rid of infeasibility (without worrying
about optimality). Once feasibility is restored, the
primal simplex can be used to find the optimum.
Alternatively, we can first apply the primal simplex to
secure optimality (without worrying about feasibility)
and then use the dual simplex to seek feasibility.
114
3.3.2 – Generalized Simplex Algorithm
Exercise 3.8
 Solve the following problem by using the generalized
simplex algorithm.
Maximize z = 2x3
subject to
-x1 + 2x2 - 2x3 ≥ 8
- x1 + x2 + x3 ≤ 4
2x1 - x2 + 4x3 ≤ 10
x1 , x2 , x3 ≥ 0
115
3.3.2 – Generalized Simplex Algorithm
Solution:
116
3.3.2 – Generalized Simplex Algorithm
 In the sensitivity analysis section, we dealt with the
sensitivity of the optimum solution by determining
the ranges for the different parameters that would
keep the optimum basic solution unchanged.
 In this section, we deal with making changes in the
parameters of the model and finding the new
optimum solution.
117
3.4 – Post-Optimal Analysis
 The following table lists the cases that can arise in
post-optimal analysis and the actions needed to obtain
the new solution (assuming one exists):
118
3.4 – Post-Optimal Analysis
Condition after parameters change Recommended action
Current solution remains optimal and
feasible.
No further action is necessary.
Current solution becomes infeasible. Use dual simplex to recover feasibility.
Current solution becomes nonoptimal.
Use primal simplex to recover
optimality.
Current solution becomes both
nonoptimal and infeasible.
Use the generalized simplex method to
obtain new solution.
 The TOYCO model of Example 3.3 will be used to
explain the different procedures. Recall that the TOYCO
model deals with the assembly of three types of. toys:
trains, trucks, and cars. Three operations are involved in
the assembly.
119
3.4 – Post-Optimal Analysis
TOYCO primal TOYCO dual
Max 𝑧 = 3𝑥1 + 2𝑥2 + 5𝑥3 Min 𝑤 = 430𝑦1 + 460𝑦2 + 420𝑦3
subject to subject to
𝑥1 + 2𝑥2 + 𝑥3 ≤ 430 (Operation 1) 𝑦1 + 3𝑦2 + 𝑦3 ≥ 3
3𝑥1 + 2𝑥3 ≤ 460 (Operation 2) 2𝑦1 + 4𝑦3 ≥ 2
𝑥1 + 4𝑥2 ≤ 420 (Operation 3) 𝑦1 + 2𝑦2 ≥ 5
𝑥1, 𝑥2, 𝑥3 ≥ 0 𝑦1, 𝑦2, 𝑦3 ≥ 0
Optimum solution: Optimum solution:
𝑥1 = 0, 𝑥2 = 100, 𝑥3 = 230, 𝑧 = $1350 𝑦1 = 1, 𝑦2 = 2, 𝑦3 = 0, 𝑤 = $1350
 The associated optimum tableau for the primal is
given as
120
3.4 – Post-Optimal Analysis
Basic X1 X2 X3 X4 X5 X6 Solution
z 4 0 0 1 2 0 1350
X2 -1/4 1 0 ½ -1/4 0 100
X3 3/2 0 1 0 ½ 0 230
X6 2 0 0 -2 1 1 20
 The feasibility of the current optimum solution may
be affected only if (1) the right-hand side of the
constraints is changed, or (2) a new constraint is
added to the model.
 In both cases, infeasibility occurs when at least one
element of the right-hand side of the optimal tableau
becomes negative-that is, one or more of the current
basic variables become negative.
121
3.4.1 – Changes Affecting Feasibility
Changes in the right-hand side.
 This change requires recomputing the right-hand side
of the tableau using Formula 1 in previous section:
 Recall that the right-hand side of the tableau gives the
values of the basic variables.
122
3.4.1 – Changes Affecting Feasibility
New RHS of
tableau in iteration 𝑖
=
Inverse in
iteration 𝑖
×
New RHS
of constraint
Example 3.11
 Situation 1. Suppose that TOYCO is increasing the daily
capacity of operations 1, 2, and 3 to 600, 640, and 590
minutes, respectively. How would this change affect the
total revenue?
 Situation 2. Although the new solution is appealing from
the standpoint of increased revenue, TOYCO recognizes
that its implementation may take time. Another proposal
was thus made to shift the slack capacity of operation 3 (x6
= 20 minutes) to the capacity of operation 1. How would
this change impact the optimum solution?
123
3.4.1 – Changes Affecting Feasibility
Solution: Situation 1.
 With these increases, the only change that will take place
in the optimum tableau is the right-hand side of the
constraints (and the optimum objective value). Thus, the
new basic solution is computed as follows:
 Thus, the current basic variables, x2, x3, and x6, remain
feasible at the new values 140, 320, and 30 units. The
associated optimum revenue is $1880.
124
3.4.1 – Changes Affecting Feasibility
𝑥2
𝑥3
𝑥6
=
1/2 −1/4 0
0 1/2 0
−2 1 1
600
640
590
=
140
320
30
Situation 2.
 The capacity mix of the three operations changes to
450,460, and 400 minutes, respectively. The resulting
solution is:
 The resulting solution is infeasible because x6 = -40,
which requires applying the dual simplex method to
recover feasibility.
125
3.4.1 – Changes Affecting Feasibility
𝑥2
𝑥3
𝑥6
=
1/2 −1/4 0
0 1/2 0
−2 1 1
450
460
400
=
110
230
−40
 First, we modify the right-hand side of the tableau as
shown by the shaded column. Notice that the associated
value of z = 3 × 0 + 2 × 110 + 5 × 230 = $1370.
126
3.4.1 – Changes Affecting Feasibility
Basic X1 X2 X3 X4 X5 X6 Solution
z 4 0 0 1 2 0 1370
X2 -1/4 1 0 ½ -1/4 0 110
X3 3/2 0 1 0 ½ 0 230
X6 2 0 0 -2 1 1 -40
 From the dual simplex, x6 leaves and x4 enters, which
yields the following optimal feasible tableau.
127
3.4.1 – Changes Affecting Feasibility
Basic X1 X2 X3 X4 X5 X6 Solution
z 5 0 0 0 5/2 1/2 1350
X2 1/4 1 0 0 0 1/4 100
X3 3/2 0 1 0 1/2 0 230
X4 -1 0 0 1 -1/2 -1/2 20
 The optimum solution (in terms of x1 , x2 , and x3)
remains the same as in the original model. This means
that the proposed shift in capacity allocation is not
advantageous because it simply shifts the surplus
capacity in operation 3 to a surplus capacity in
operation 1. The conclusion is that operation 2 is the
bottleneck and it may be advantageous to shift the
surplus to operation 2 instead. The selection of
operation 2 over operation 1 is also reinforced by the
fact that the dual price for operation 2 ($2/min) is
higher than that for operation 1 ( $l/min).
128
3.4.1 – Changes Affecting Feasibility
 This section considers two particular situations that
could affect the optimality of the current solution:
1. Changes in the original objective coefficients.
2. Addition of a new economic activity (variable) to
the model.
129
3.4.2 – Changes Affecting Optimality
Changes in the Objective Function Coefficients.
 These changes affect only the optimality of the
solution and requires recomputing the z-row
coefficients according to the following procedure:
1. Compute the dual values using Method 2 in
previous section.
2. Use the new dual values in Formula 2, to determine
the new z-row coefficients.
130
3.4.2 – Changes Affecting Optimality
 Two cases will result:
1. New z-row satisfies the optimality condition. The
solution remains unchanged (the optimum objective
value may change, however).
2. The optimality condition is not satisfied. Apply the
primal simplex method to recover optimality.
131
3.4.2 – Changes Affecting Optimality
Example 3.13
 Situation 1. In the TOYCO model, suppose that the
company has a new pricing policy to meet the
competition. The unit revenues under the new policy
are $2, $3, and $4 for train, truck, and car toys,
respectively. How is the optimal solution affected?
132
3.4.2 – Changes Affecting Optimality
Solution:
 The new objective function is
Maximize z = 2x1 + 3x2 + 4x3
Thus,
(New objective coefficients of x2, x3, and x6) = (3, 4, 0)
Using Method 2, the dual variables are computed as
133
3.4.2 – Changes Affecting Optimality
(𝑦1 𝑦2 𝑦3) = (3 4 0)
1/2 −1/4 0
0 1/2 0
−2 1 1
= (3/2 5/4 0)
 The z-row coefficients are determined as the
difference between the left- and right-hand sides of
the dual constraints (Fonnula 2). It is not necessary to
recompute the objective-row coefficients of the basic
variables x2, x3, and x6 because they always equal zero
regardless of any changes made in the objective
coefficients.
134
3.4.2 – Changes Affecting Optimality
Obj. coeff. of 𝑥1 = 𝑦1 + 3𝑦2 + 𝑦3 − 𝟐 = 13/4
Obj. coeff. of 𝑥4 = 𝑦1 − 0 = 3/2
Obj. coeff. of 𝑥5 = 𝑦2 − 0 = 5/4
 Note that the right-hand side of the first dual
constraint is 2, the new coefficient in the modified
objective function.
 The computations show that the current solution,
x1 = 0 train, x2 = 100 trucks, and x3 = 230 cars,
remains optimal. The corresponding new revenue is
computed as 2 × 0 + 3 × 100 + 4 × 230 = $1220. The
new pricing policy is not advantageous because it
leads to lower revenue.
135
3.4.2 – Changes Affecting Optimality
 Situation 2. Suppose now that the TOYCO objective
function is changed to
Maximize z = 6x1 + 3x2 + 4x3
 We have
136
3.4.2 – Changes Affecting Optimality
(𝑦1 𝑦2 𝑦3) = (3 4 0)
1/2 −1/4 0
0 1/2 0
−2 1 1
= (3/2 5/4 0)
Obj. coeff. of 𝑥1 = 𝑦1 + 3𝑦2 + 𝑦3 − 𝟔 = −3/4
Obj. coeff. of 𝑥4 = 𝑦1 − 0 = 3/2
Obj. coeff. of 𝑥5 = 𝑦2 − 0 = 5/4
 The new reduced cost of x1 shows that the current solution
is not optimum.
To determine the new solution, the z-row is changed as
highlighted in the following tableau:
137
3.4.2 – Changes Affecting Optimality
Basic X1 X2 X3 X4 X5 X6 Solution
z -3/4 0 0 3/2 5/4 0 1220
X2 -1/4 1 0 ½ -1/4 0 100
X3 3/2 0 1 0 ½ 0 230
X6 2 0 0 -2 1 1 20
 The highlighted elements are the new objective
coefficients and new objective value. All the
remaining elements are the same as in the original
optimal tableau. The new optimum solution is then
determined by letting x1 enter and x6 leave, which
yields
x1 = 10, x2 = 102.5, x3 = 215, and z = $1227.50.
 Although the new solution recommends the
production of all three toys, the optimum revenue is
less than when two toys only are manufactured.
138
3.4.2 – Changes Affecting Optimality
Exercise 3.9:
 Consider the following LP:
 a) Compute the entire simplex tableau associated with
139
.
0
,
,
3
2
1
.
.
2
max
3
2
1
2
1
3
2
3
1
3
2
1










x
x
x
x
x
x
x
x
x
t
s
x
x
x
z
Basic variables = (𝑥1 , 𝑥2 , 𝑥3) and Inverse =
1
2
−
1
2
1
2
−
1
2
1
2
1
2
1
2
1
2
−
1
2
 b) check for feasibility and optimality of solution from
part (a).
 c) Find the range of values of c1 (the contribution to
profit of x1) for which the current basis remains
optimal, assuming that all other parameters remain
unchanged.
 d) Find the range of values of b3 (the right hand side of
constraint 1) for which the current basis remains
optimal, assuming that all other parameters remain
unchanged.
140
The End
141

More Related Content

What's hot

FORECASTING ERRORS (2) (2).pptx
FORECASTING ERRORS (2) (2).pptxFORECASTING ERRORS (2) (2).pptx
FORECASTING ERRORS (2) (2).pptxPradipDulal2
 
Goal Programming
Goal ProgrammingGoal Programming
Goal ProgrammingEvren E
 
LINEAR PROGRAMMING Assignment help
LINEAR PROGRAMMING Assignment helpLINEAR PROGRAMMING Assignment help
LINEAR PROGRAMMING Assignment helpjohn mayer
 
Game theory and its applications
Game theory and its applicationsGame theory and its applications
Game theory and its applicationsEranga Weerasekara
 
North west corner method
North west corner method North west corner method
North west corner method Akash Jadhav
 
Decision making under uncertainty
Decision making under uncertaintyDecision making under uncertainty
Decision making under uncertaintySagar Khairnar
 
primal and dual problem
primal and dual problemprimal and dual problem
primal and dual problemYash Lad
 
Game theory application
Game theory applicationGame theory application
Game theory applicationshakebaumar
 
Linear programming Cost Minimization
Linear programming Cost MinimizationLinear programming Cost Minimization
Linear programming Cost MinimizationKhushbu :-)
 
Quadratic programming (Tool of optimization)
Quadratic programming (Tool of optimization)Quadratic programming (Tool of optimization)
Quadratic programming (Tool of optimization)VARUN KUMAR
 
Quantity discount
Quantity discountQuantity discount
Quantity discountHriday Bora
 
Inventory model (operation research)
Inventory model (operation research)Inventory model (operation research)
Inventory model (operation research)Amrita Gharti
 
Newsvendor news boy
Newsvendor news boy Newsvendor news boy
Newsvendor news boy FayshalMiazy
 
Dynamic Programming
Dynamic ProgrammingDynamic Programming
Dynamic Programmingcontact2kazi
 
Unit.3. duality and sensetivity analisis
Unit.3. duality and sensetivity analisisUnit.3. duality and sensetivity analisis
Unit.3. duality and sensetivity analisisDagnaygebawGoshme
 

What's hot (20)

FORECASTING ERRORS (2) (2).pptx
FORECASTING ERRORS (2) (2).pptxFORECASTING ERRORS (2) (2).pptx
FORECASTING ERRORS (2) (2).pptx
 
Goal Programming
Goal ProgrammingGoal Programming
Goal Programming
 
LINEAR PROGRAMMING Assignment help
LINEAR PROGRAMMING Assignment helpLINEAR PROGRAMMING Assignment help
LINEAR PROGRAMMING Assignment help
 
Duality
DualityDuality
Duality
 
Game theory and its applications
Game theory and its applicationsGame theory and its applications
Game theory and its applications
 
North west corner method
North west corner method North west corner method
North west corner method
 
Linear programming
Linear programmingLinear programming
Linear programming
 
Game theory
Game theoryGame theory
Game theory
 
Decision making under uncertainty
Decision making under uncertaintyDecision making under uncertainty
Decision making under uncertainty
 
simplex method
simplex methodsimplex method
simplex method
 
primal and dual problem
primal and dual problemprimal and dual problem
primal and dual problem
 
Game theory application
Game theory applicationGame theory application
Game theory application
 
Linear programming Cost Minimization
Linear programming Cost MinimizationLinear programming Cost Minimization
Linear programming Cost Minimization
 
Linear programming
Linear programmingLinear programming
Linear programming
 
Quadratic programming (Tool of optimization)
Quadratic programming (Tool of optimization)Quadratic programming (Tool of optimization)
Quadratic programming (Tool of optimization)
 
Quantity discount
Quantity discountQuantity discount
Quantity discount
 
Inventory model (operation research)
Inventory model (operation research)Inventory model (operation research)
Inventory model (operation research)
 
Newsvendor news boy
Newsvendor news boy Newsvendor news boy
Newsvendor news boy
 
Dynamic Programming
Dynamic ProgrammingDynamic Programming
Dynamic Programming
 
Unit.3. duality and sensetivity analisis
Unit.3. duality and sensetivity analisisUnit.3. duality and sensetivity analisis
Unit.3. duality and sensetivity analisis
 

Similar to Chapter_3-_Sensitivity-duality_-_students.pptx

Linear programming models - U2.pptx
Linear programming models - U2.pptxLinear programming models - U2.pptx
Linear programming models - U2.pptxMariaBurgos55
 
CH-2 Linear Programing.pptx
CH-2 Linear Programing.pptxCH-2 Linear Programing.pptx
CH-2 Linear Programing.pptxRobiel1
 
Proyecto parcial ii_grupo2.docx
Proyecto parcial ii_grupo2.docxProyecto parcial ii_grupo2.docx
Proyecto parcial ii_grupo2.docxLuisCuevaFlores
 
Chapter 2 Linear Programming for business (1).pptx
Chapter 2 Linear Programming for business (1).pptxChapter 2 Linear Programming for business (1).pptx
Chapter 2 Linear Programming for business (1).pptxanimutsileshe1
 
Operation cost reduction in unit commitment problem using improved quantum bi...
Operation cost reduction in unit commitment problem using improved quantum bi...Operation cost reduction in unit commitment problem using improved quantum bi...
Operation cost reduction in unit commitment problem using improved quantum bi...IJECEIAES
 
211658558-Chapter-4-Economic-Dispatch.pdf
211658558-Chapter-4-Economic-Dispatch.pdf211658558-Chapter-4-Economic-Dispatch.pdf
211658558-Chapter-4-Economic-Dispatch.pdfyoziscijunior
 
HYBRID PARTICLE SWARM OPTIMIZATION FOR SOLVING MULTI-AREA ECONOMIC DISPATCH P...
HYBRID PARTICLE SWARM OPTIMIZATION FOR SOLVING MULTI-AREA ECONOMIC DISPATCH P...HYBRID PARTICLE SWARM OPTIMIZATION FOR SOLVING MULTI-AREA ECONOMIC DISPATCH P...
HYBRID PARTICLE SWARM OPTIMIZATION FOR SOLVING MULTI-AREA ECONOMIC DISPATCH P...ijsc
 
Bis 311 final examination answers
Bis 311 final examination answersBis 311 final examination answers
Bis 311 final examination answersRandalHoffman
 
Hybrid Particle Swarm Optimization for Solving Multi-Area Economic Dispatch P...
Hybrid Particle Swarm Optimization for Solving Multi-Area Economic Dispatch P...Hybrid Particle Swarm Optimization for Solving Multi-Area Economic Dispatch P...
Hybrid Particle Swarm Optimization for Solving Multi-Area Economic Dispatch P...ijsc
 
Influence of Emissions Trading Scheme on Electricity Markets and Prices
Influence of Emissions Trading Scheme on Electricity Markets and PricesInfluence of Emissions Trading Scheme on Electricity Markets and Prices
Influence of Emissions Trading Scheme on Electricity Markets and PricesNicola Asker
 
IRJET- Comparison for Max-Flow Min-Cut Algorithms for Optimal Assignment Problem
IRJET- Comparison for Max-Flow Min-Cut Algorithms for Optimal Assignment ProblemIRJET- Comparison for Max-Flow Min-Cut Algorithms for Optimal Assignment Problem
IRJET- Comparison for Max-Flow Min-Cut Algorithms for Optimal Assignment ProblemIRJET Journal
 
3 Introduction To Linear Programming
3 Introduction To Linear Programming3 Introduction To Linear Programming
3 Introduction To Linear ProgrammingDawn Cook
 
The optimal solution for unit commitment problem using binary hybrid grey wol...
The optimal solution for unit commitment problem using binary hybrid grey wol...The optimal solution for unit commitment problem using binary hybrid grey wol...
The optimal solution for unit commitment problem using binary hybrid grey wol...IJECEIAES
 
IJERD(www.ijerd.com)International Journal of Engineering Research and Develop...
IJERD(www.ijerd.com)International Journal of Engineering Research and Develop...IJERD(www.ijerd.com)International Journal of Engineering Research and Develop...
IJERD(www.ijerd.com)International Journal of Engineering Research and Develop...IJERD Editor
 
IJERD (www.ijerd.com) International Journal of Engineering Research and Devel...
IJERD (www.ijerd.com) International Journal of Engineering Research and Devel...IJERD (www.ijerd.com) International Journal of Engineering Research and Devel...
IJERD (www.ijerd.com) International Journal of Engineering Research and Devel...IJERD Editor
 
Mr. Bakar Presentation for group discussion.docx
Mr. Bakar Presentation for group discussion.docxMr. Bakar Presentation for group discussion.docx
Mr. Bakar Presentation for group discussion.docxCyberMohdSalahShoty
 
DAM assignment - LPP formulation, Graphical solution and Simplex Method
DAM assignment - LPP formulation, Graphical solution and Simplex MethodDAM assignment - LPP formulation, Graphical solution and Simplex Method
DAM assignment - LPP formulation, Graphical solution and Simplex MethodNeha Kumar
 

Similar to Chapter_3-_Sensitivity-duality_-_students.pptx (20)

P2 ch03 2
P2 ch03 2P2 ch03 2
P2 ch03 2
 
Linear programming models - U2.pptx
Linear programming models - U2.pptxLinear programming models - U2.pptx
Linear programming models - U2.pptx
 
CH-2 Linear Programing.pptx
CH-2 Linear Programing.pptxCH-2 Linear Programing.pptx
CH-2 Linear Programing.pptx
 
Proyecto parcial ii_grupo2.docx
Proyecto parcial ii_grupo2.docxProyecto parcial ii_grupo2.docx
Proyecto parcial ii_grupo2.docx
 
Chapter 2 Linear Programming for business (1).pptx
Chapter 2 Linear Programming for business (1).pptxChapter 2 Linear Programming for business (1).pptx
Chapter 2 Linear Programming for business (1).pptx
 
Operation cost reduction in unit commitment problem using improved quantum bi...
Operation cost reduction in unit commitment problem using improved quantum bi...Operation cost reduction in unit commitment problem using improved quantum bi...
Operation cost reduction in unit commitment problem using improved quantum bi...
 
211658558-Chapter-4-Economic-Dispatch.pdf
211658558-Chapter-4-Economic-Dispatch.pdf211658558-Chapter-4-Economic-Dispatch.pdf
211658558-Chapter-4-Economic-Dispatch.pdf
 
HYBRID PARTICLE SWARM OPTIMIZATION FOR SOLVING MULTI-AREA ECONOMIC DISPATCH P...
HYBRID PARTICLE SWARM OPTIMIZATION FOR SOLVING MULTI-AREA ECONOMIC DISPATCH P...HYBRID PARTICLE SWARM OPTIMIZATION FOR SOLVING MULTI-AREA ECONOMIC DISPATCH P...
HYBRID PARTICLE SWARM OPTIMIZATION FOR SOLVING MULTI-AREA ECONOMIC DISPATCH P...
 
Bis 311 final examination answers
Bis 311 final examination answersBis 311 final examination answers
Bis 311 final examination answers
 
Hybrid Particle Swarm Optimization for Solving Multi-Area Economic Dispatch P...
Hybrid Particle Swarm Optimization for Solving Multi-Area Economic Dispatch P...Hybrid Particle Swarm Optimization for Solving Multi-Area Economic Dispatch P...
Hybrid Particle Swarm Optimization for Solving Multi-Area Economic Dispatch P...
 
linear programming
linear programming linear programming
linear programming
 
Logistics Management Homework Help
Logistics Management Homework HelpLogistics Management Homework Help
Logistics Management Homework Help
 
Influence of Emissions Trading Scheme on Electricity Markets and Prices
Influence of Emissions Trading Scheme on Electricity Markets and PricesInfluence of Emissions Trading Scheme on Electricity Markets and Prices
Influence of Emissions Trading Scheme on Electricity Markets and Prices
 
IRJET- Comparison for Max-Flow Min-Cut Algorithms for Optimal Assignment Problem
IRJET- Comparison for Max-Flow Min-Cut Algorithms for Optimal Assignment ProblemIRJET- Comparison for Max-Flow Min-Cut Algorithms for Optimal Assignment Problem
IRJET- Comparison for Max-Flow Min-Cut Algorithms for Optimal Assignment Problem
 
3 Introduction To Linear Programming
3 Introduction To Linear Programming3 Introduction To Linear Programming
3 Introduction To Linear Programming
 
The optimal solution for unit commitment problem using binary hybrid grey wol...
The optimal solution for unit commitment problem using binary hybrid grey wol...The optimal solution for unit commitment problem using binary hybrid grey wol...
The optimal solution for unit commitment problem using binary hybrid grey wol...
 
IJERD(www.ijerd.com)International Journal of Engineering Research and Develop...
IJERD(www.ijerd.com)International Journal of Engineering Research and Develop...IJERD(www.ijerd.com)International Journal of Engineering Research and Develop...
IJERD(www.ijerd.com)International Journal of Engineering Research and Develop...
 
IJERD (www.ijerd.com) International Journal of Engineering Research and Devel...
IJERD (www.ijerd.com) International Journal of Engineering Research and Devel...IJERD (www.ijerd.com) International Journal of Engineering Research and Devel...
IJERD (www.ijerd.com) International Journal of Engineering Research and Devel...
 
Mr. Bakar Presentation for group discussion.docx
Mr. Bakar Presentation for group discussion.docxMr. Bakar Presentation for group discussion.docx
Mr. Bakar Presentation for group discussion.docx
 
DAM assignment - LPP formulation, Graphical solution and Simplex Method
DAM assignment - LPP formulation, Graphical solution and Simplex MethodDAM assignment - LPP formulation, Graphical solution and Simplex Method
DAM assignment - LPP formulation, Graphical solution and Simplex Method
 

More from SIAWSINGONGKPMGuru

kertas1-120711215546-phpapp02.ppt
kertas1-120711215546-phpapp02.pptkertas1-120711215546-phpapp02.ppt
kertas1-120711215546-phpapp02.pptSIAWSINGONGKPMGuru
 
TOPIC_5_b_-_Islamic_Bkg_Ops_Instruments_Retail_only_.pptx
TOPIC_5_b_-_Islamic_Bkg_Ops_Instruments_Retail_only_.pptxTOPIC_5_b_-_Islamic_Bkg_Ops_Instruments_Retail_only_.pptx
TOPIC_5_b_-_Islamic_Bkg_Ops_Instruments_Retail_only_.pptxSIAWSINGONGKPMGuru
 
NOTA-MATEMATIK-edited-by-MAZIAH.ppt
NOTA-MATEMATIK-edited-by-MAZIAH.pptNOTA-MATEMATIK-edited-by-MAZIAH.ppt
NOTA-MATEMATIK-edited-by-MAZIAH.pptSIAWSINGONGKPMGuru
 
Kuliah Motivasi M7 PART 1 & 2 3oct2017 kelas.ppt
Kuliah Motivasi M7 PART 1 & 2 3oct2017 kelas.pptKuliah Motivasi M7 PART 1 & 2 3oct2017 kelas.ppt
Kuliah Motivasi M7 PART 1 & 2 3oct2017 kelas.pptSIAWSINGONGKPMGuru
 
Group_4-_Gender_and_learning_meta_analysis.pptx
Group_4-_Gender_and_learning_meta_analysis.pptxGroup_4-_Gender_and_learning_meta_analysis.pptx
Group_4-_Gender_and_learning_meta_analysis.pptxSIAWSINGONGKPMGuru
 
Kajian_Meta_Analisis__Penggunaan_ICT_oleh_guru_dalam_PdP_Bahasa_Melayu_1.pptx
Kajian_Meta_Analisis__Penggunaan_ICT_oleh_guru_dalam_PdP_Bahasa_Melayu_1.pptxKajian_Meta_Analisis__Penggunaan_ICT_oleh_guru_dalam_PdP_Bahasa_Melayu_1.pptx
Kajian_Meta_Analisis__Penggunaan_ICT_oleh_guru_dalam_PdP_Bahasa_Melayu_1.pptxSIAWSINGONGKPMGuru
 

More from SIAWSINGONGKPMGuru (15)

Bandingan-semacam.pptx
Bandingan-semacam.pptxBandingan-semacam.pptx
Bandingan-semacam.pptx
 
kertas1-120711215546-phpapp02.ppt
kertas1-120711215546-phpapp02.pptkertas1-120711215546-phpapp02.ppt
kertas1-120711215546-phpapp02.ppt
 
TOPIC_5_b_-_Islamic_Bkg_Ops_Instruments_Retail_only_.pptx
TOPIC_5_b_-_Islamic_Bkg_Ops_Instruments_Retail_only_.pptxTOPIC_5_b_-_Islamic_Bkg_Ops_Instruments_Retail_only_.pptx
TOPIC_5_b_-_Islamic_Bkg_Ops_Instruments_Retail_only_.pptx
 
Bahagikan Biskut.pptx
Bahagikan Biskut.pptxBahagikan Biskut.pptx
Bahagikan Biskut.pptx
 
tatabahasa__imbuhan_.pptx
tatabahasa__imbuhan_.pptxtatabahasa__imbuhan_.pptx
tatabahasa__imbuhan_.pptx
 
羽毛球.pptx
羽毛球.pptx羽毛球.pptx
羽毛球.pptx
 
01_BCSJK_DSKP_Y3.pptx
01_BCSJK_DSKP_Y3.pptx01_BCSJK_DSKP_Y3.pptx
01_BCSJK_DSKP_Y3.pptx
 
NOTA-MATEMATIK-edited-by-MAZIAH.ppt
NOTA-MATEMATIK-edited-by-MAZIAH.pptNOTA-MATEMATIK-edited-by-MAZIAH.ppt
NOTA-MATEMATIK-edited-by-MAZIAH.ppt
 
波波.ppt
波波.ppt波波.ppt
波波.ppt
 
Module_1-Overview.pptx
Module_1-Overview.pptxModule_1-Overview.pptx
Module_1-Overview.pptx
 
Bahasa Melayu.pptx
Bahasa Melayu.pptxBahasa Melayu.pptx
Bahasa Melayu.pptx
 
Kuliah Motivasi M7 PART 1 & 2 3oct2017 kelas.ppt
Kuliah Motivasi M7 PART 1 & 2 3oct2017 kelas.pptKuliah Motivasi M7 PART 1 & 2 3oct2017 kelas.ppt
Kuliah Motivasi M7 PART 1 & 2 3oct2017 kelas.ppt
 
Group_4-_Gender_and_learning_meta_analysis.pptx
Group_4-_Gender_and_learning_meta_analysis.pptxGroup_4-_Gender_and_learning_meta_analysis.pptx
Group_4-_Gender_and_learning_meta_analysis.pptx
 
Kajian_Meta_Analisis__Penggunaan_ICT_oleh_guru_dalam_PdP_Bahasa_Melayu_1.pptx
Kajian_Meta_Analisis__Penggunaan_ICT_oleh_guru_dalam_PdP_Bahasa_Melayu_1.pptxKajian_Meta_Analisis__Penggunaan_ICT_oleh_guru_dalam_PdP_Bahasa_Melayu_1.pptx
Kajian_Meta_Analisis__Penggunaan_ICT_oleh_guru_dalam_PdP_Bahasa_Melayu_1.pptx
 
单元二:我爱设计.pptx
单元二:我爱设计.pptx单元二:我爱设计.pptx
单元二:我爱设计.pptx
 

Recently uploaded

HMCS Vancouver Pre-Deployment Brief - May 2024 (Web Version).pptx
HMCS Vancouver Pre-Deployment Brief - May 2024 (Web Version).pptxHMCS Vancouver Pre-Deployment Brief - May 2024 (Web Version).pptx
HMCS Vancouver Pre-Deployment Brief - May 2024 (Web Version).pptxmarlenawright1
 
Fostering Friendships - Enhancing Social Bonds in the Classroom
Fostering Friendships - Enhancing Social Bonds  in the ClassroomFostering Friendships - Enhancing Social Bonds  in the Classroom
Fostering Friendships - Enhancing Social Bonds in the ClassroomPooky Knightsmith
 
Towards a code of practice for AI in AT.pptx
Towards a code of practice for AI in AT.pptxTowards a code of practice for AI in AT.pptx
Towards a code of practice for AI in AT.pptxJisc
 
On_Translating_a_Tamil_Poem_by_A_K_Ramanujan.pptx
On_Translating_a_Tamil_Poem_by_A_K_Ramanujan.pptxOn_Translating_a_Tamil_Poem_by_A_K_Ramanujan.pptx
On_Translating_a_Tamil_Poem_by_A_K_Ramanujan.pptxPooja Bhuva
 
FSB Advising Checklist - Orientation 2024
FSB Advising Checklist - Orientation 2024FSB Advising Checklist - Orientation 2024
FSB Advising Checklist - Orientation 2024Elizabeth Walsh
 
Jamworks pilot and AI at Jisc (20/03/2024)
Jamworks pilot and AI at Jisc (20/03/2024)Jamworks pilot and AI at Jisc (20/03/2024)
Jamworks pilot and AI at Jisc (20/03/2024)Jisc
 
Interdisciplinary_Insights_Data_Collection_Methods.pptx
Interdisciplinary_Insights_Data_Collection_Methods.pptxInterdisciplinary_Insights_Data_Collection_Methods.pptx
Interdisciplinary_Insights_Data_Collection_Methods.pptxPooja Bhuva
 
Basic Intentional Injuries Health Education
Basic Intentional Injuries Health EducationBasic Intentional Injuries Health Education
Basic Intentional Injuries Health EducationNeilDeclaro1
 
Single or Multiple melodic lines structure
Single or Multiple melodic lines structureSingle or Multiple melodic lines structure
Single or Multiple melodic lines structuredhanjurrannsibayan2
 
ICT Role in 21st Century Education & its Challenges.pptx
ICT Role in 21st Century Education & its Challenges.pptxICT Role in 21st Century Education & its Challenges.pptx
ICT Role in 21st Century Education & its Challenges.pptxAreebaZafar22
 
On National Teacher Day, meet the 2024-25 Kenan Fellows
On National Teacher Day, meet the 2024-25 Kenan FellowsOn National Teacher Day, meet the 2024-25 Kenan Fellows
On National Teacher Day, meet the 2024-25 Kenan FellowsMebane Rash
 
Wellbeing inclusion and digital dystopias.pptx
Wellbeing inclusion and digital dystopias.pptxWellbeing inclusion and digital dystopias.pptx
Wellbeing inclusion and digital dystopias.pptxJisc
 
Beyond_Borders_Understanding_Anime_and_Manga_Fandom_A_Comprehensive_Audience_...
Beyond_Borders_Understanding_Anime_and_Manga_Fandom_A_Comprehensive_Audience_...Beyond_Borders_Understanding_Anime_and_Manga_Fandom_A_Comprehensive_Audience_...
Beyond_Borders_Understanding_Anime_and_Manga_Fandom_A_Comprehensive_Audience_...Pooja Bhuva
 
NO1 Top Black Magic Specialist In Lahore Black magic In Pakistan Kala Ilam Ex...
NO1 Top Black Magic Specialist In Lahore Black magic In Pakistan Kala Ilam Ex...NO1 Top Black Magic Specialist In Lahore Black magic In Pakistan Kala Ilam Ex...
NO1 Top Black Magic Specialist In Lahore Black magic In Pakistan Kala Ilam Ex...Amil baba
 
Salient Features of India constitution especially power and functions
Salient Features of India constitution especially power and functionsSalient Features of India constitution especially power and functions
Salient Features of India constitution especially power and functionsKarakKing
 
Kodo Millet PPT made by Ghanshyam bairwa college of Agriculture kumher bhara...
Kodo Millet  PPT made by Ghanshyam bairwa college of Agriculture kumher bhara...Kodo Millet  PPT made by Ghanshyam bairwa college of Agriculture kumher bhara...
Kodo Millet PPT made by Ghanshyam bairwa college of Agriculture kumher bhara...pradhanghanshyam7136
 
SOC 101 Demonstration of Learning Presentation
SOC 101 Demonstration of Learning PresentationSOC 101 Demonstration of Learning Presentation
SOC 101 Demonstration of Learning Presentationcamerronhm
 
Tatlong Kwento ni Lola basyang-1.pdf arts
Tatlong Kwento ni Lola basyang-1.pdf artsTatlong Kwento ni Lola basyang-1.pdf arts
Tatlong Kwento ni Lola basyang-1.pdf artsNbelano25
 
Philosophy of china and it's charactistics
Philosophy of china and it's charactisticsPhilosophy of china and it's charactistics
Philosophy of china and it's charactisticshameyhk98
 

Recently uploaded (20)

HMCS Vancouver Pre-Deployment Brief - May 2024 (Web Version).pptx
HMCS Vancouver Pre-Deployment Brief - May 2024 (Web Version).pptxHMCS Vancouver Pre-Deployment Brief - May 2024 (Web Version).pptx
HMCS Vancouver Pre-Deployment Brief - May 2024 (Web Version).pptx
 
Fostering Friendships - Enhancing Social Bonds in the Classroom
Fostering Friendships - Enhancing Social Bonds  in the ClassroomFostering Friendships - Enhancing Social Bonds  in the Classroom
Fostering Friendships - Enhancing Social Bonds in the Classroom
 
Towards a code of practice for AI in AT.pptx
Towards a code of practice for AI in AT.pptxTowards a code of practice for AI in AT.pptx
Towards a code of practice for AI in AT.pptx
 
On_Translating_a_Tamil_Poem_by_A_K_Ramanujan.pptx
On_Translating_a_Tamil_Poem_by_A_K_Ramanujan.pptxOn_Translating_a_Tamil_Poem_by_A_K_Ramanujan.pptx
On_Translating_a_Tamil_Poem_by_A_K_Ramanujan.pptx
 
FSB Advising Checklist - Orientation 2024
FSB Advising Checklist - Orientation 2024FSB Advising Checklist - Orientation 2024
FSB Advising Checklist - Orientation 2024
 
Jamworks pilot and AI at Jisc (20/03/2024)
Jamworks pilot and AI at Jisc (20/03/2024)Jamworks pilot and AI at Jisc (20/03/2024)
Jamworks pilot and AI at Jisc (20/03/2024)
 
Interdisciplinary_Insights_Data_Collection_Methods.pptx
Interdisciplinary_Insights_Data_Collection_Methods.pptxInterdisciplinary_Insights_Data_Collection_Methods.pptx
Interdisciplinary_Insights_Data_Collection_Methods.pptx
 
Basic Intentional Injuries Health Education
Basic Intentional Injuries Health EducationBasic Intentional Injuries Health Education
Basic Intentional Injuries Health Education
 
Single or Multiple melodic lines structure
Single or Multiple melodic lines structureSingle or Multiple melodic lines structure
Single or Multiple melodic lines structure
 
ICT Role in 21st Century Education & its Challenges.pptx
ICT Role in 21st Century Education & its Challenges.pptxICT Role in 21st Century Education & its Challenges.pptx
ICT Role in 21st Century Education & its Challenges.pptx
 
On National Teacher Day, meet the 2024-25 Kenan Fellows
On National Teacher Day, meet the 2024-25 Kenan FellowsOn National Teacher Day, meet the 2024-25 Kenan Fellows
On National Teacher Day, meet the 2024-25 Kenan Fellows
 
Wellbeing inclusion and digital dystopias.pptx
Wellbeing inclusion and digital dystopias.pptxWellbeing inclusion and digital dystopias.pptx
Wellbeing inclusion and digital dystopias.pptx
 
Beyond_Borders_Understanding_Anime_and_Manga_Fandom_A_Comprehensive_Audience_...
Beyond_Borders_Understanding_Anime_and_Manga_Fandom_A_Comprehensive_Audience_...Beyond_Borders_Understanding_Anime_and_Manga_Fandom_A_Comprehensive_Audience_...
Beyond_Borders_Understanding_Anime_and_Manga_Fandom_A_Comprehensive_Audience_...
 
NO1 Top Black Magic Specialist In Lahore Black magic In Pakistan Kala Ilam Ex...
NO1 Top Black Magic Specialist In Lahore Black magic In Pakistan Kala Ilam Ex...NO1 Top Black Magic Specialist In Lahore Black magic In Pakistan Kala Ilam Ex...
NO1 Top Black Magic Specialist In Lahore Black magic In Pakistan Kala Ilam Ex...
 
Salient Features of India constitution especially power and functions
Salient Features of India constitution especially power and functionsSalient Features of India constitution especially power and functions
Salient Features of India constitution especially power and functions
 
Kodo Millet PPT made by Ghanshyam bairwa college of Agriculture kumher bhara...
Kodo Millet  PPT made by Ghanshyam bairwa college of Agriculture kumher bhara...Kodo Millet  PPT made by Ghanshyam bairwa college of Agriculture kumher bhara...
Kodo Millet PPT made by Ghanshyam bairwa college of Agriculture kumher bhara...
 
SOC 101 Demonstration of Learning Presentation
SOC 101 Demonstration of Learning PresentationSOC 101 Demonstration of Learning Presentation
SOC 101 Demonstration of Learning Presentation
 
Mehran University Newsletter Vol-X, Issue-I, 2024
Mehran University Newsletter Vol-X, Issue-I, 2024Mehran University Newsletter Vol-X, Issue-I, 2024
Mehran University Newsletter Vol-X, Issue-I, 2024
 
Tatlong Kwento ni Lola basyang-1.pdf arts
Tatlong Kwento ni Lola basyang-1.pdf artsTatlong Kwento ni Lola basyang-1.pdf arts
Tatlong Kwento ni Lola basyang-1.pdf arts
 
Philosophy of china and it's charactistics
Philosophy of china and it's charactisticsPhilosophy of china and it's charactistics
Philosophy of china and it's charactistics
 

Chapter_3-_Sensitivity-duality_-_students.pptx

  • 1.
  • 2.  In LP, the parameters (input data) of the model can change within certain limits without causing the optimum solution to change. This is referred to as sensitivity analysis.  Dual analysis deals with determining the new optimum solution resulting from making targeted changes in the input data. 3.1 – Sensitivity Analysis 2
  • 3.  In this section two cases will be considered. 1. Sensitivity of the optimum solution to changes in the availability of the resources (right-hand side of the constraints). 2. Sensitivity of the optimum solution to changes in unit profit or unit cost (coefficients of the objective function). 3 3.1.1 – Graphical Sensitivity Analysis
  • 4. Example 3.1 (Changes in the right hand side)  JOBCO produces two products on two machines. A unit of product 1 requires 2 hours on machine 1 and 1 hour on machine 2. For product 2, a unit requires 1 hour on machine 1 and 3 hours on machine 2. The revenues per unit of products 1 and 2 are $30 and $20, respectively. The total daily processing time available for each machine is 8 hours.  Let X1 and X2 represent the daily number of units of products 1 and 2, respectively. 4 3.1.1 – Graphical Sensitivity Analysis
  • 5. Solution:  The LP model is given as Maximize z = 30X1 + 20X2 s.t. 2X1 + X2 ≤ 8 (Machine 1) X1 + 3X2 ≤ 8 (Machine 2) X1 , X2 ≥ 0 -----  Figure 3.1 illustrates the change in the optimum solution when changes are made in the capacity of machine 1. 5 3.1.1 – Graphical Sensitivity Analysis
  • 6.  If the daily capacity is increased from 8 hours to 9 hours, the new optimum will occur at point G. 6 3.1.1 – Graphical Sensitivity Analysis Figure 3.1
  • 7.  The rate of change in optimum z resulting from changing machine 1 capacity from 8 hours to 9 hours can be computed as follows:  The computed rate provides a direct link between the model input (resources) and its output (total revenue). It says that a unit increase (decrease) in machine 1 capacity will increase (decrease) revenue by $14. 7 3.1.1 – Graphical Sensitivity Analysis Rate of revenue change resulting from incresing machine 1 capacity by 1 hr (point C to point G) = 𝑧𝐺 − 𝑧𝐶 (capacity change) = 142 − 128 9 − 8 = $14 /hr
  • 8.  The name unit worth of a resource is an apt description of the rate of change of the objective function per unit change of a resource. Now it is called dual (or shadow) price.  In Fig. 3.1 it can be seen that the dual price of $14.00/hr remains valid for changes (increases or decreases) in machine 1 capacity that move its constraint parallel to itself to any point on the line segment BF. This means that the range of applicability of the given dual price can be computed as follows: 8 3.1.1 – Graphical Sensitivity Analysis
  • 9. Minimum machine 1 capacity [at B = (0, 2.67)] = 2 × 0 + 1 × 2.67 = 2.67 hr Maximum machine 1 capacity [at F = (8, 0)] = 2 × 8 + 1 × 0 = 16 hr  The conclusion is that the dual price of $14/hr will remain valid for the range 2.67 hr ≤ Machine 1 capacity ≤ 16 hr Changes outside this range will produce a different dual price (worth per unit). 9 3.1.1 – Graphical Sensitivity Analysis
  • 10. Minimum machine 1 capacity [at B = (0, 2.67)] = 2 × 0 + 1 × 2.67 = 2.67 hr Maximum machine 1 capacity [at F = (8, 0)] = 2 × 8 + 1 × 0 = 16 hr 10 3.1.1 – Graphical Sensitivity Analysis Capacity of M1 𝟐𝒙𝟏 + 𝒙𝟐 Z-value 𝟑𝟎𝒙𝟏 + 𝟐𝟎𝒙𝟐 𝐵 = (0, 2.67) 2 0 + 8/3 = 8/3 30 0 + 20 8/3 = 160/3 𝐹 = (8, 0) 2 8 + 0 = 16 30 8 + 20 0 = 240 Dual price = ∆ 𝑧_value ∆ capacity 𝑀1 = 240 − 160/3 16 − 8/3 = $14/ℎ𝑟 feasible range: 2.67 ≤ capacity of 𝑀1 ≤ 16
  • 11.  By using similar computations, the dual price for machine 2 capacity is $2.00/hr and it remains valid for changes (increases or decreases) that move its constraint parallel to itself to any point on the line segment DE, which yields the following limits: 11 3.1.1 – Graphical Sensitivity Analysis Capacity of M2 𝒙𝟏 + 𝟑𝒙𝟐 Z-value 𝟑𝟎𝒙𝟏 + 𝟐𝟎𝒙𝟐 D = (4, 0) 4 + 3 0 = 4 30 4 + 20 0 = 120 E = (0, 8) 0 + 3 8 = 24 30 0 + 20 8 = 160 Dual price = ∆ 𝑧_value ∆ capacity 𝑀2 = 160 − 120 24 − 4 = $2/ℎ𝑟 feasibility range: 4 ≤ capacity of 𝑀2 ≤ 24
  • 12.  The conclusion is that the dual price of $2.00/hr for machine 2 will remain applicable for the range 4 hr ≤ Machine 2 capacity ≤ 24 hr  The computed limits for machine 1 and 2 are referred to as the feasibility ranges.  The dual prices allow making economic decisions about the LP problem, as the following questions demonstrate: 12 3.1.1 – Graphical Sensitivity Analysis
  • 13.  Question 1. If JOBCO can increase the capacity of both machines, which machine should receive priority?  Question 2. A suggestion is made to increase the capacities of machines 1 and 2 at the additional cost of $10/hr. Is this advisable? 13 3.1.1 – Graphical Sensitivity Analysis
  • 14.  Question 3. If the capacity of machine 1 is increased from the present 8 hours to 13 hours, how will this increase impact the optimum revenue? 14 3.1.1 – Graphical Sensitivity Analysis
  • 15.  Question 4. Suppose that the capacity of machine 1 is increased to 20 hours, how will this increase impact the optimum revenue?  Remember that falling outside the feasibility range does not mean that the problem has no solution. It only means that available information is not sufficient to make a complete decision. 15 3.1.1 – Graphical Sensitivity Analysis
  • 16.  Question 5. The change in the optimum objective value equals (dual price × change in resource) so long as the change in the resource is within the feasibility range. How can we determine the new optimum values of the variables associated with a change in a resource? 16 3.1.1 – Graphical Sensitivity Analysis
  • 17. Example 3.2 (Changes in the objective coefficient)  Fig. 3.2 shows the graphical solution space of the JOBCO problem in Example 3.1. The optimum occurs at point C (X1 = 3.2, X2 = 1.6, Z = 128). Changes in revenue units (i.e., objective-function coefficients) will change the slope of z. However, as can be seen from the figure, the optimum solution at point C remains unchanged as long as the objective function lies between lines BF and DE, the two constraints that define the optimum point. 17 3.1.1 – Graphical Sensitivity Analysis
  • 18.  This means that there is a range for the coefficients of the objective function that will keep the optimum solution unchanged at C. 18 3.1.1 – Graphical Sensitivity Analysis Figure 3.2
  • 19.  To determine ranges for the coefficients of the objective function that keep the optimum solution unchanged at point C , first write the objective function can in the general format Maximize z = c1 X1 + c2 X2 Imagine now that the line z is pivoted at C and that it can rotate clockwise and counterclockwise.  The optimum solution will remain at point C so long as z = c1 X1 + c2 X2 lies between the two lines X1 + 3X2 =8 and 2X1 + X2 = 8. 19 3.1.1 – Graphical Sensitivity Analysis
  • 20.  This means that the ratio c1/c2 can vary between 1/3 and 2/1 , which yields the following optimality range.  This information can provide immediate answers regarding the optimum solution as the following questions demonstrate: 20 3.1.1 – Graphical Sensitivity Analysis 1 3 ≤ 𝑐1 𝑐2 ≤ 2 1 or 0.333 ≤ 𝑐1 𝑐2 ≤ 2
  • 21.  Question 1. Suppose that the unit revenues for products 1 and 2 are changed to $35 and $25, respectively. Will the current optimum remain the same? 21 3.1.1 – Graphical Sensitivity Analysis
  • 22.  Question 2. Suppose that the unit revenue of product 2 is fixed at its current value c2 = $20. What is the associated optimality range for the unit revenue for product 1, c1 , that will keep the optimum unchanged? 22 3.1.1 – Graphical Sensitivity Analysis
  • 23. Exercise 3.1:  Wild West produces two types of cowboy hats. A Type 1 hat requires twice as much labor time as a Type 2. If all the available labor time is dedicated to Type 2 alone, the company can produce a total of 400 Type 2 hats a day. The respective market limits for the two types are 150 and 200 hats per day. The revenue is $8 per Type 1 hat and $5 per Type 2 hat. a) Use the graphical solution to determine the number of hats of each type that maximizes revenue. 23 3.1.1 – Graphical Sensitivity Analysis
  • 24. b) Determine the dual price of the production capacity (in terms of the Type 2 hat) and the range for which it is applicable. c) If the daily demand limit on the Type 1 hat is decreased to 120, use the dual price to determine the corresponding effect on the optimal revenue. d) What is the dual price of the market share of the Type 2 hat? By how much can the market share be increased while yielding the computed worth per unit? e) Determine the optimality range for the unit revenue ratio of the two types of hats that will keep the current optimum unchanged. f) Using the information in (e), will the optimal solution change if the revenue per unit is the same for both types? 24 3.1.1 – Graphical Sensitivity Analysis
  • 25. Solution: 25 3.1.1 – Graphical Sensitivity Analysis
  • 26. 26 3.1.1 – Graphical Sensitivity Analysis
  • 27.  This section extends the sensitivity analysis to the general LP model to determine the dual price by using algebraic calculation.  A numeric example (the TOYCO model) will be used to facilitate the presentation. 27 3.1.2 – Algebraic Sensitivity Analysis
  • 28. Example 3.3 (TOYCO Model)  TOYCO uses three operations to assemble three types of toys-trains, trucks, and cars. The daily available times for the three operations are 430, 460, and 420 minutes, respectively, and the revenues per unit of toy train, truck, and car are $3, $2, and $5, respectively. The assembly times per train at the three operations are 1, 3, and 1 minutes, respectively. The corresponding times per train and per car are (2,0,4) and (1,2,0) minutes (a zero time indicates that the operation is not used). 28 3.1.2 – Algebraic Sensitivity Analysis
  • 29. Solution:  Letting X1 , X2 , and X3 represent the daily number of units assembled of trains, trucks, and cars, respectively, the associated LP model is given as: Maximize z = 3X1 + 2X2 + 5X3 s.t. X1 + 2X2 + X3 ≤ 430 (operation 1) 3X1 + 2X3 ≤ 460 (operation 2) X1 + 4X2 ≤ 420 (operation 3) X1 , X2 , X3 ≥ 0 (operation 1)  Using X4 , X5 , and X6 as the slack variables 29 3.1.2 – Algebraic Sensitivity Analysis
  • 30.  The optimal tableau is  The solution recommends manufacturing 100 trucks and 230 cars but no trains. The associated revenue is $1350. 30 3.1.2 – Algebraic Sensitivity Analysis Basic X1 X2 X3 X4 X5 X6 Solution z 4 0 0 1 2 0 1350 X2 -1/4 1 0 ½ -1/4 0 100 X3 3/2 0 1 0 ½ 0 230 X6 2 0 0 -2 1 1 20
  • 31. Determination of Dual Prices and feasibility ranges  Recognizing that the dual prices and their feasibility ranges are rooted in making changes in the right-hand side of the constraints, suppose that D1 , D2 , and D3 are the (positive or negative) changes made in the allotted daily manufacturing time of operations 1, 2, and 3, respectively.  The original TOYCO model can then be changed to 31 3.1.2 – Algebraic Sensitivity Analysis
  • 32. Maximize z = 3X1 + 2X2 + 5X3 s.t. X1 + 2X2 + X3 ≤ 430 + D1 (operation 1) 3X1 + 2X3 ≤ 460 + D2 (operation 2) X1 + 4X2 ≤ 420 + D3 (operation 3) X1 , X2 , X3 ≥ 0 (operation 1)  First rewrite the starting tableau using the new right-hand side, 430 + D1 , 460 + D2 , and 420 + D3 . 32 3.1.2 – Algebraic Sensitivity Analysis
  • 33.  The two shaded areas are identical. Hence, if we repeat the same simplex iterations as in the original model, the columns in the two groups will also be identical in the optimal tableau. 33 3.1.2 – Algebraic Sensitivity Analysis Solution Basic X1 X2 X3 X4 X5 X6 RHS D1 D2 D3 z -3 -2 -5 0 0 0 0 0 0 0 X4 1 2 1 1 0 0 430 1 0 0 X5 3 0 2 0 1 0 460 0 1 0 X6 1 4 0 0 0 1 420 0 0 1
  • 34.  That is 34 3.1.2 – Algebraic Sensitivity Analysis Solution Basic X1 X2 X3 X4 X5 X6 RHS D1 D2 D3 z 4 0 0 1 2 0 1350 1 2 0 X2 -1/4 1 0 ½ -1/4 0 100 ½ -1/4 0 X3 3/2 0 1 0 ½ 0 230 0 ½ 0 X6 2 0 0 -2 1 1 20 -2 1 1
  • 35.  The new optimum tableau provides the following optimal solution: z = 1350 + D1 + 2D2 X2 = 100 + 1/2D1 – 1/4D2 X3 = 230 + 1/2D2 X6 = 20 - 2D1 + D2 + D3  Now use this solution to determine the dual prices and the feasibility ranges. 35 3.1.2 – Algebraic Sensitivity Analysis
  • 36. Dual prices:  The value of the objective function can be written as z = 1350 + 1D1 + 2D2 + 0D3  This means that, by definition, the corresponding dual prices are 1, 2, and 0 ($/min) for operations 1, 2, and 3, respectively.  The coefficients of D1 , D2 , and D3 in the optimal z-row are exactly those of the slack variables X4 , X5 , and X6. This means that the dual prices equal the coefficients of the slack variables in the optimal z-row. 36 3.1.2 – Algebraic Sensitivity Analysis
  • 37. Feasibility range:  The current solution remains feasible if all the basic variables remain nonnegative: X2 = 100 + 1/2D1 – 1/4D2 ≥ 0 X3 = 230 + 1/2D2 ≥ 0 X6 = 20 - 2D1 + D2 + D3 ≥ 0  Simultaneous changes D1 , D2 , and D3 that satisfy these inequalities will keep the solution feasible 37 3.1.2 – Algebraic Sensitivity Analysis
  • 38.  Suppose that the manufacturing time available for operations 1, 2, and 3 are 480, 440, and 400 minutes respectively. Then D1 =480–430= 50, D2=440 –460=-20, D3=400–420= -20 Substituting in the feasibility conditions X2 = 100 + ½(50)– ¼(-20)= 130 > 0  Feasible X3 = 230 + ½(-20)= 220 > 0  Feasible X6 = 20 – 2(50) + (-20) + (-20)= -120 < 0  Infeasible 38 3.1.2 – Algebraic Sensitivity Analysis
  • 39.  If D1 = -30, D2= –12, D3= 10 , then X2 = 100 + ½(-30)– ¼(-12)= 88 > 0  Feasible X3 = 230 + ½(-12)= 224 > 0  Feasible X6 = 20 – 2(-30) + (-12) + (10)= 78 < 0  Feasible The new optimal solution is X2 = 88, X3 = 224, X6 = 78 with z = 1350 + 1(-30) + 2(-12) + 0(10) = $1296.  The given conditions can produce the individual feasibility ranges associated with changing the resources one at a time. 39 3.1.2 – Algebraic Sensitivity Analysis
  • 40.  For example, a change of time only for operation 1 means that D2 = D3 = 0. So, X2 = 100 + 1/2D1 ≥ 0 X3 = 230 ≥ 0  -200 ≤ D1 ≤ 10 X6 = 20 - 2D1 ≥ 0 Similarly, -20 ≤ D2 ≤ 400 and -20 ≤ D3 < ∞ 40 3.1.2 – Algebraic Sensitivity Analysis Resource amount (minutes) Resource Dual price ($) Feasibility range Minimum Current Maximum Operation 1 1 -200 ≤ D1 ≤ 10 230 430 440 Operation 2 2 -20 ≤ D2 ≤ 400 440 460 860 Operation 3 0 -20 ≤ D3 < ∞ 400 420 ∞
  • 41. Exercise 3.2:  For problem in exercise 1 find dual price and feasibility ranges. Maximize z = 8X1 + 5X2 s.t. 2X1 + X2 ≤ 400 + D1 X1 ≤ 150 + D2 X2 ≤ 200 + D3 X1 , X2 ≥ 0 41 3.1.2 – Algebraic Sensitivity Analysis
  • 42. Solution: 42 3.1.2 – Algebraic Sensitivity Analysis
  • 43. Determination of the optimality ranges - Changes in objective coefficients  In the TOYCO model, let d1 , d2 , and d3 represent the change in unit revenues for toy trucks, trains, and cars, respectively. Maximize z = 3X1 + 2X2 + 5X3 s.t. X1 + 2X2 + X3 ≤ 430 (operation 1) 3X1 + 2X3 ≤ 460 (operation 2) X1 + 4X2 ≤ 420 (operation 3) X1 , X2 , X3 ≥ 0 (operation 1) 43 3.1.2 – Algebraic Sensitivity Analysis
  • 44.  The objective function then becomes Maximize z = (3+ d1)X1 + (2+ d2)X2 + (5+ d3)X3  With the simultaneous changes, the z-row in the starting tableau appears as: 44 3.1.2 – Algebraic Sensitivity Analysis Basic X1 X2 X3 X4 X5 X6 Solution z -3- d1 -2- d2 -5- d3 0 0 0 0 X4 1 2 1 1 0 0 430 X5 3 0 2 0 1 0 460 X6 1 4 0 0 0 1 420
  • 45.  When we generate the simplex tableaus using the same sequence of entering and leaving variables in the original model (before the changes dj are introduced), the optimal iteration will appear as follows 45 3.1.2 – Algebraic Sensitivity Analysis Basic X1 X2 X3 X4 X5 X6 Solution z 4-1/4d2 +3/2d3-d1 0 0 1+1/2d2 2-1/4d2 +1/2d3 0 1350+100d2 +230d3 X2 -1/4 1 0 ½ -1/4 0 100 X3 3/2 0 1 0 ½ 0 230 X6 2 0 0 -2 1 1 20
  • 46.  The new optimal tableau is the same as in the original optimal tableau except for the z-equation coefficients. This means that changes in the objective-function coefficients can affect the optimality of the problem only. ( Changes in the right-hand side affect feasibility only)  An examination of the new z-row shows that the coefficients of dj are taken directly from the constraint coefficients of the optimum tableau. A convenient way for computing is to add a new top row and a new leftmost column to the optimum tableau, as shown by the shaded areas below. 46 3.1.2 – Algebraic Sensitivity Analysis
  • 47.  The entries in the top row are the change dj associated with variable Xj. For the leftmost column, the top element is 1 in the z-row followed by change dj for basic variable Xj. Keep in mind that dj = 0 for the slack variables. z = (3+ d1)X1 + (2+ d2)X2 + (5+ d3)X3 47 3.1.2 – Algebraic Sensitivity Analysis d1 d2 d3 0 0 0 Basic X1 X2 X3 X4 X5 X6 Solution 1 z 4 0 0 1 2 0 1350 d2 X2 -1/4 1 0 ½ -1/4 0 100 d3 X3 3/2 0 1 0 ½ 0 230 0 X6 2 0 0 -2 1 1 20
  • 48.  To compute the new objective coefficients of nonbasic variables (or the value of z), multiply the elements of its column by the corresponding elements in the leftmost column, add them up, and subtract the top- row element from the sum. For example, for X1 , we have Objective coefficient of X1 = [4 × 1 + (-1/4) × d2 + (3/2) × d3 + 2 × 0] – d1 = 4 – 1/4 d2 + 3/2 d3 - d1 48 3.1.2 – Algebraic Sensitivity Analysis
  • 49.  The current solution remains optimal so long as the new z- equation coefficients remain nonnegative (maximization case). We thus have the following simultaneous optimality conditions corresponding to nonbasic X1 , X4 , and X5 : 4 – 1/4 d2 + 3/2 d3 - d1 ≥ 0 1 + 1/2 d2 ≥ 0 2 – 1/4 d2 + 1/2 d3 ≥ 0  Remember that the z-equation coefficient for a basic variable is always zero. 49 3.1.2 – Algebraic Sensitivity Analysis
  • 50.  To illustrate the use of these conditions, suppose that the objective function of TOYCO is changed from z = 3X1 + 2X2 + 5X3 to z = 2X1 + X2 + 6X3 . Then, d1 = 2 – 3 = -$1 , d2 = 1 – 2 = -$1 and d3 = 6 – 5 = $1 substitution in the given conditions yields 4 – 1/4 d2 + 3/2 d3 - d1 = 4 –1/4 (-1)+3/2 (1)- (-1)= 6.75 > 0 1 + 1/2 d2 = 1 + ½ (-1) = 0.5 > 0 2 – 1/4 d2 + 1/2 d3 = 2 – ¼ (-1) + ½ (1) = 2.75 > 0 all are satisfied. 50 3.1.2 – Algebraic Sensitivity Analysis
  • 51.  The results show that the proposed changes will keep the current solution (X1 = 0, X2 = 100, X3 = 230) optimal (with a new value of z = 1350 + 100d2 + 230d3 = 1350 + 100(-1)+ 230(1) = $1480 If any of the conditions is not satisfied, a new solution must be determined.  The only difference in the minimization case is that the z- equations coefficients must be ≤ 0 to maintain optimality. 51 3.1.2 – Algebraic Sensitivity Analysis
  • 52.  The optimality ranges dealing with changing dj one at a time can be developed from the simultaneous optimality conditions. For example, suppose that the objective coefficient of X2 only is changed to 2 + d2 meaning that d1 = d3 = 0. the simultaneous optimality conditions thus reduce to 4 – 1/4 d2 ≥ 0  d2 ≤ 16 1 + 1/2 d2 ≥ 0  d2 ≥ -2  -2 ≤ d2 ≤ 8 2 – 1/4 d2 ≥ 0  d2 ≤ 8 Also, d1 < 4 and d3 ≥ -8/3 52 3.1.2 – Algebraic Sensitivity Analysis
  • 53.  The given individual conditions can be translated to total unit revenue ranges. For example, for toy trucks (variable X2), the total unit revenue is 2 + d2 and its optimality range -2 ≤ d2 ≤ 8 translates to 2 -2 ≤ 2+d2 ≤ 2+8 $0 ≤ c2 (unit revenue of toy truck) ≤ $10  It assumes that the unit revenues for toy trains and toy cars remain fixed at $3 and $5, respectively. 53 3.1.2 – Algebraic Sensitivity Analysis
  • 54.  It is important to notice that the changes d1 , d2 , and d3 may be within their allowable individual ranges without satisfying the simultaneous conditions, and vice versa. For example, consider z = 6X1 + 8X2 + 3X3 . Here, d1 = 6-3= 3, d2 = 8-2= 6, and d3 = 3-5= -2 (-∞ < d1 < 4 , -2 ≤ d2 ≤ 8 , and -8/3 ≤ d3 < ∞) However, the corresponding simultaneous conditions yield 4 – 1/4 d2 + 3/2 d3 - d1 = 4 –1/4 (6)+3/2 (-2)- 3= -3.5 < 0 1 + 1/2 d2 = 1 + ½ (6) = 4 > 0 2 – 1/4 d2 + 1/2 d3 = 2 – ¼ (6) + ½ (-2) = -0.5 < 0 54 3.1.2 – Algebraic Sensitivity Analysis
  • 55. Exercise 3.3:  HiDec produces two models of electronic gadgets that use resistors, capacitors, and chips. The following table summarizes the data of the situation: 55 3.1.2 – Algebraic Sensitivity Analysis Unit resource requirements Resource Model 1 (units) Model 2 (units) Maximum availability (units) Resistor 2 3 1200 Capacitor 2 1 1000 Chips 0 4 800 Unit price ($) 3 4
  • 56. a) Determine the status of each resource. b) In terms of the optimal revenue, determine the dual prices for the resistors, capacitors, and chips. c) Determine the feasibility ranges for the dual prices obtained in (b). d) If the available number of resistors is increased to 1300 units, find the new optimum solution. e) If the available number of chips is reduced to 350 units, will you be able to determine the new optimum solution directly from the given information? Explain. 56 3.1.2 – Algebraic Sensitivity Analysis
  • 57. f) If the availability of capacitors is limited by the feasibility range computed in (c), determine the corresponding range of the optimal revenue and the corresponding ranges for the numbers of units to be produced of Models 1 and 2. g) A new contractor is offering to sell HiDec additional resistors at 40 cents each, but only if HiDec would purchase at least 500 units. Should HiDec accept the offer? 57 3.1.2 – Algebraic Sensitivity Analysis
  • 58. Solution: 58 3.1.2 – Algebraic Sensitivity Analysis
  • 59. 59 3.1.2 – Algebraic Sensitivity Analysis
  • 60. Exercise 3.4:  The Bank of Elkins is allocating a maximum of $200,000 for personal and car loans during the next month. The bank charges 14% for personal loans and 12% for car loans. Both types of loans are repaid at the end of a 1-year period. Experience shows that about 3% of personal loans and 2% of car loans are not repaid. The bank usually allocates at least twice as much to car loans as to personal loans. a) Determine the optimal allocation of funds between the two loans and the net rate of return on all the loans. b) If the percentages of personal and car loans are changed to 4% and 3%, respectively, use sensitivity analysis to determine jf the optimum solution in (a) will change. 60 3.1.2 – Algebraic Sensitivity Analysis
  • 61. Solution: 61 3.1.2 – Algebraic Sensitivity Analysis
  • 62. 62 3.1.2 – Algebraic Sensitivity Analysis
  • 63.  The dual problem is defined systematically from the primal (or original) LP model. The two problems are closely related, in the sense that the optimal solution of one problem automatically provides the optimal solution to the other.  In most LP treatments, the dual is defined for various forms of the primal depending on the sense of optimization (maximization or minimization), types of constraints (≤, ≥, or =), and sign of the variables (nonnegative or unrestricted). 63 3.2 – Duality
  • 64.  The present definition of the dual problem requires expressing the primal problem in the equation form (all the constraints are equations with nonnegative right-hand side and all the variables are nonnegative).  This requirement is consistent with the format of the simplex starting tableau. Hence, any results obtained from the primal optimal solution will apply directly to the associated dual problem. 64 3.2 – Duality
  • 65.  Steps for constructing the dual from the primal are : 1) Assign a dual variable for each primal (equality) constraint. 2) Construct a dual constraint for each primal variable. 3) The column (constraint) coefficients and the objective coefficient of the jth primal variable respectively define the left-hand and the right-hand sides of the jth dual constraint. 4) The dual objective coefficients equal the right-hand sides of the primal constraint equations. 65 3.2 – Duality
  • 66.  The sense of optimization, direction of inequalities, and the signs of the variables in the dual are in following table. 66 3.2 – Duality Dual problem Primal problem objective Objective Constrains type Variable sign Maximization Minimization ≥ Unrestricted Minimization Maximization ≤ Unrestricted
  • 67. Example 3.4  Primal model: Maximize z = 5X1 + 12X2 + 4X3 S.t. X1 + 2X2 + X3 ≤ 10 2X1 – X2 + 3X3 = 8 X1 , X2 , X3 ≥ 0  Equation form: Max z = 5X1 + 12X2 + 4X3 + 0X4 S.t. Dual variables X1 + 2X2 + X3 + X4 = 10  y1 2X1 – X2 + 3X3 + 0X4 = 8  y2 X1 , X2 , X3 , X4 ≥ 0 67 3.2 – Duality
  • 68.  Dual problem Minimize w = 10y1 + 8y2 S.t. y1 + 2y2 ≥ 5 2y1 - y2 ≥ 12 y1 + 3y2 ≥ 4 y1 + 0y2 ≥ 0 y1 , y2 unrestricted  y1 ≥ 0 , y2 unrestricted 68 3.2 – Duality
  • 69. Example 3.5  Primal model: Minimize z = 15X1 + 12X2 S.t. X1 + 2X2 ≥ 3 2X1 – 4X2 ≤ 5 X1 , X2 ≥ 0  Equation form: 69 3.2 – Duality
  • 71. Example 3.6  Primal model: Maximize z = 5X1 + 6X2 S.t. X1 + 2X2 = 5 -X1 + 5X2 ≥ 3 4X1 + 7X2 ≤ 8 X1 unrestricted, X2 ≥ 0 71 3.2 – Duality
  • 73.  The first and second constraints are replaced by an equation. The general rule in this case is that an unrestricted primal variable always corresponds to an equality dual constraint. Conversely, a primal equation produces an unrestricted dual variable, as the first primal constraint demonstrates.  Note: No provision is made for including artificial variables in the primal, because artificial variables would not change the definition of the dual problem. 73 3.2 – Duality
  • 74.  The following table summarizes the primal-dual rules: 74 3.2 – Duality Maximization problem Minimization problem Constraints Variables ≥  ≤ 0 ≤  ≥ 0 =  Unrestricted Variables Constraints ≥ 0  ≥ ≤ 0  ≤ Unrestricted  =
  • 75. 3.2.1 - Primal-Dual Relationships  Changes made in the original LP model can affect the optimality and/or the feasibility of the current solution. This section introduces a number of primal- dual relationships that can be used to recompute the elements of the optimal simplex tableau. 75 3.2 – Duality
  • 76. Starting Variables Objective z-row = Constraint columns 1 0 … 0 0 1 … 0 ⁞ ⁞ … 0 = 0 0 … 1 Identity matrix (Starting Tableau) 76 2.2 – The Simplex Method Figure 3.4 (a)
  • 77. 77 2.2 – The Simplex Method Starting Variables Objective z-row = Constraint columns Inverse matrix = (General Iteration) Figure 3.4 (b)
  • 78.  Figure 3.4 gives a schematic representation of the starting and general simplex tableaus. In the starting tableau, the constraint coefficients under the starting variables form an identity matrix (all main-diagonal elements equal 1 and all off-diagonal elements equal zero). With this arrangement, subsequent iterations of the simplex tableau generated by the Gauss-Jordan row operations will modify the elements of the identity matrix to produce what is known as the inverse matrix. The inverse matrix is key to computing all the elements of the associated simplex tableau. 78 3.2.2 – Simplex Tableau Layout
  • 79. 3.2.3 – Optimal Dual Solution  The primal and dual solutions are closely related that the optimal solution of either problem directly yields the optimal solution to the other. Thus, in an LP model in which the number of variables is considerably smaller than the number of constraints, computational savings may be realized by solving the dual because the amount of simplex computation depends largely (though not totally) on the number of constraints. 79 3.2 – Duality
  • 80. This section provides two methods for determining the dual values.  Method 1.  Method 2. 80 3.2.3 – Optimal Dual Solution Optimal values of 𝑑𝑢𝑎𝑙 variables = Row vector of original objective coefficients of optimal 𝑝𝑟𝑖𝑚𝑎𝑙 basic variables × Optimal 𝑝𝑟𝑖𝑚𝑎𝑙 inverse Optimal value of dual variable yi = Optimal primal z-coefficient of starting variable xi + original objective coefficient of xi
  • 81.  The elements of the row vector must appear in the same order the basic variables are listed in the Basic column of the simplex tableau. 81 3.2.3 – Optimal Dual Solution
  • 82. Example 3.7  Consider the following LP: Maximize z = 5X1 + 12X2 + 4X3 S.t. X1 + 2X2 + X3 ≤ 10 2X1 – X2 + 3X3 = 8 X1 , X2 , X3 ≥ 0  Primal: Max z = 5X1 + 12X2 + 4X3 - MR S.t. Dual variables X1 + 2X2 + X3 + X4 = 10  y1 2X1 – X2 + 3X3 + R = 8  y2 X1 , X2 , X3 , X4 , R ≥ 0 82 3.2.3 – Optimal Dual Solution
  • 83.  Dual problem Minimize w = 10y1 + 8y2 S.t. y1 + 2y2 ≥ 5 2y1 - y2 ≥ 12 y1 + 3y2 ≥ 4 y1 ≥ 0 y2 ≥ -M  y2 unrestricted 83 3.2.3 – Optimal Dual Solution
  • 84.  Optimal tableau of the primal  In this table the starting primal variables X4 and R uniquely correspond to the dual variables yl and y2 , respectively. Thus, we determine the optimum dual solution as follows: 84 3.2.3 – Optimal Dual Solution Basic X1 X2 X3 X4 R Solution z 0 0 3/5 29/5 -2/5+ M 274/5 X2 0 1 -1/5 2/5 -1/5 12/5 X1 1 0 7/5 1/5 2/5 26/5
  • 85.  Method 1. 85 3.2.3 – Optimal Dual Solution Starting primal basic variables X4 R Z-equation coefficients 29/5 -2/5+ M Original objective coefficient 0 -M Dual variables y1 y2 Optimal dual values 29/5 + 0 = 29/5 -2/5 +M+(-M)= -2/5
  • 86.  Method 2. The optimal inverse matrix, highlighted in the table under the starting variables X4 and R, is The order of the optimal primal basic variables in the Basic column is X2 followed by X1. The elements of the original objective coefficients for the two variables must appear in the same order-namely, 86 3.2.3 – Optimal Dual Solution optimal inverse = 2/5 −1/5 1/5 2/5
  • 87. (Original objective coefficients)=coefficient of (X2 , X1) = (12, 5)  The optimal values are 87 3.2.3 – Optimal Dual Solution (𝑦1 𝑦2) = (12 5) × 2 5 − 1 5 1 5 2 5 = (29/5 −2/5)
  • 88. Primal-Dual objective values:  For any pair of feasible primal and dual solutions,  At the optimum, the relationship holds as a strict equation. The relationship does not specify which problem is primal and which is dual. Only the sense of optimization (maximization or minimization) is important in this case. 88 3.2.3 – Optimal Dual Solution Objective value in the 𝑚𝑎𝑥𝑖𝑚𝑖𝑧𝑎𝑡𝑖𝑜𝑛 problem ≤ Objective value in the 𝑚𝑖𝑛𝑖𝑚𝑖𝑧𝑎𝑡𝑖𝑜𝑛 problem Figure 3.5
  • 89. Example 3.8  In Example 3.7, (X1 = 0, X2 = 0, X3 = 8/3) and ( y1 = 6, y2 = 0) are (arbitrary) feasible primal and dual solutions. The associated values of the objective functions are z = 5X1 + 12X2 + 4X3 = 5(0) + 12(0) + 4(8/3) = 32/3 w = 10y1 + 8y2 = 10(6) + 8(0) = 60 Thus, z (=32/3) for the maximization problem (primal) is less than w (=60) for the minimization problem (dual). The optimum value of z (= 274/5) falls within the range (32/3, 60). 89 3.2.3 – Optimal Dual Solution
  • 90. Exercise 3.5:  Estimate a range for the optimal objective value for the following LP: Minimize z = 5X1 + 2X2 S.t. X1 - X2 ≥ 3 2X1 + 3X2 ≥ 5 X1 , X2 ≥ 0 90 3.2.3 – Optimal Dual Solution
  • 91. Solution:  Dual problem: 91 3.2.3 – Optimal Dual Solution
  • 92. 3.2.4 - Simplex Tableau Computations  This section shows how any iteration of the entire simplex tableau can be generated from the original data of the problem, the inverse associated with the iteration, and the dual problem. Using the layout of the simplex tableau in Figure 3.4, we can divide the computations into two types: 1) Constraint columns (left- and right-hand sides). 2) Objective z-row. 92 3.2 – Duality
  • 93. 93 Figure 3.4 3.2.4 – Simplex Tableau Computations
  • 94.  Formula 1: Constraint Column Computations. In any simplex iteration, a left-hand or a right-hand side column is computed as follows:  Formula 2: Objective z-row Computations. In any simplex iteration, the objective equation coefficient of xi is computed as follows: 94 3.2.4 – Simplex Tableau Computations Constraint column in iteration 𝑖 = Inverse in iteration 𝑖 × Original constraint column Primal z-equation coefficient of variable xi = Left-hand side of jth dual constraint – Right-hand side of jth dual constraint .
  • 95. Example 3.9  Consider the LP in Example 3.7, use the formulas 1 and 2 and find the coefficient of X1 in optimum tableau. Maximize z = 5X1 + 12X2 + 4X3 S.t. X1 + 2X2 + X3 ≤ 10 2X1 – X2 + 3X3 = 8 X1 , X2 , X3 ≥ 0 95 3.2.4 – Simplex Tableau Computations
  • 96. Solution:  From the table 96 3.2.4 – Simplex Tableau Computations Basic X1 X2 X3 X4 R Solution z 0 0 3/5 29/5 -2/5+ M 274/5 X2 0 1 -1/5 2/5 -1/5 12/5 X1 1 0 7/5 1/5 2/5 26/5 optimal inverse = 2/5 −1/5 1/5 2/5
  • 97.  Formula 1: Similar computation can be used to generate the optimal columns for X2, X3, X4, and R. 97 3.2.4 – Simplex Tableau Computations 𝑥1column in optimal iteration = Inverse in optimal iteration × Original x1column = 2/5 −1/5 1/5 2/5 × 1 2 = 0 1
  • 98.  Formula 2: The optimal values of the dual variables, (y1, y2)= (29/5, - 2/5), are used in formula 2 to compute all the z- coefficients. z-coefficient of X1 = LHS of constraint 1 – RHS z-coefficient of X1 = y1 + 2y2 – 5 = 29/5+ 2(-2/5)- 5 = 0 z-coefficient of R = y2 – (-M) = -2/5 – (-M) = -2/5 +M Similar computation can be used to determine the z-coefficients for X2, X3, and X4 . 98 3.2.4 – Simplex Tableau Computations
  • 99. Exercise 3.6  Consider the following LP: Minimize z = 2X1 + X2 S.t. 3X1 + X2 - X3 = 3 4X1 + 3X2 – X4 = 6 X1 + 2X2 + X5 = 3 X1 , X2 , X3 , X4 , X5 ≥ 0 99 3.2.4 – Simplex Tableau Computations
  • 100.  Compute the entire simplex tableau associated with the following basic solution and check it for optimality and feasibility. Basic variables = (X1 , X2 , X5 ), Inverse = 100 3.2.4 – Simplex Tableau Computations 3 5 − 1 5 0 − 4 5 3 5 0 1 −1 1
  • 101. Solution: 101 3.2.4 – Simplex Tableau Computations
  • 102.  This section presents two additional algorithms: The dual simplex and the generalized simplex. The dual simplex starts infeasible (but better than optimal) and remains infeasible until feasibility is restored at the last iteration.  The generalized simplex combines both the primal and dual simplex methods, starting both nonoptimal and infeasible. At the final iteration, the solution becomes optimal and feasible (assuming that one exists). 102 3.3 – Additional Simplex Algorithm
  • 103.  When we use the simplex method to solve a max problem, we begin with an initial solution that is primal feasible (nonnegative rhs) but dual infeasible (at least one variable in the z-row has a negative coefficient). Eventually, we will obtain an optimal solution that is both primal feasible and dual feasible (nonnegative z-row).  When we use the dual simplex method to solve a max problem, we begin with an initial solution that is dual feasible (nonnegative z-row) but primal infeasible (at least one constraint has a negative rhs). Eventually, we will obtain an optimal solution that is both dual feasible and primal feasible (nonnegative rhs). 103 3.3.1 – Dual Simplex Algorithm
  • 104.  Dual feasibility condition. The leaving variable, xr is the basic variable having the most negative value (ties are broken arbitrarily). If all the basic variables are nonnegative, the algorithm ends.  Dual optimality condition. Given that xr is the leaving variable, let be the objective coefficient of nonbasic variable xj and αrj the constraint coefficient in the xr-row and xj-column of the tableau. The entering variable is the nonbasic variable with αrj < 0 that corresponds to 104 3.3.1 – Dual Simplex Algorithm 𝑐𝑗 min Nonbasic 𝑥𝑗 𝑐𝑗 𝛼𝑟𝑗 , 𝛼𝑟𝑗 < 0
  • 105. (Ties are broken arbitrarily.) If αrj ≥ 0 for all nonbasic xj , the problem has no feasible solution.  To start the LP optimal and infeasible, two requirements must be met: 1. The objective function must satisfy the optimality condition of the regular simplex method. 2. All the constraints must be of the type (≤). Inequalities of the type (≥) are converted to (≤) by multiplying both sides of the inequality by -1. If the LP includes (=) constraints, the equation can be replaced by two inequalities. 105 3.3.1 – Dual Simplex Algorithm
  • 106. Example 3.10 Minimize z = 3x1 + 2x2 + x3 subject to 3x1 + 2x2 + x3 ≥ 3 -3x1 + 3x2 + x3 ≥ 6 x1 + x2 + x3 ≤ 3 x1 , x2 , x3 ≥ 0 In the present example, the first two inequalities are multiplied by -1 to convert them to (≤) constraints. 106 3.3.1 – Dual Simplex Algorithm
  • 107.  The starting tableau is thus given as:  The tableau is optimal because all the objective coefficients in the z-row are nonnegative. It is also infeasible because at least one of the basic variables is negative (x4 = -3, x5 = -6, x6 = 3). 107 3.3.1 – Dual Simplex Algorithm Basic x1 x2 x3 x4 x5 x6 Solution z -3 -2 -1 0 0 0 0 x4 -3 -2 -1 1 0 0 -3 x5 3 -3 -1 0 1 0 -6 x6 1 1 1 0 0 1 3
  • 108.  According to the dual feasibility condition, x5 (= -6) is the leaving variable. The next table shows how the dual optimality condition is used to determine the entering variable.  The ratios show that x2 is the entering variable. 108 3.3.1 – Dual Simplex Algorithm j = 1 j = 2 j = 3 Nonbasic variable x1 x2 x3 z-row -3 -2 -1 x5-row, α5j 3 -3 -1 Ratio, | / α5j |, α5j <0 - 2/3 1 𝑐𝑗 𝑐𝑗
  • 109.  The next tableau is obtained by using the familiar row operations, which gives:  Which is both optimal and feasible. 109 3.3.1 – Dual Simplex Algorithm Basic x1 x2 x3 x4 x5 x6 Solution z -5 0 -1/3 0 -2/3 0 4 x4 -5 0 -1/3 1 -2/3 0 1 x2 -1 1 1/3 0 -1/3 0 2 x6 2 0 2/3 0 1/3 1 1
  • 110.  Notice how the dual simplex works. In all the iterations, optimality is maintained (all coefficients are ≤o).At the same time, each new iteration moves the solution toward feasibility. 110 3.3.1 – Dual Simplex Algorithm
  • 111. Exercise 3.7  Solve the following problem by using the dual simplex algorithm. Minimize z = 4x1 + 2x2 subject to x1 + x2 = 10 3x1 - x2 ≥ 20 x1 , x2 ≥ 0 111 3.3.1 – Dual Simplex Algorithm
  • 112. Solution: 112 3.3.1 – Dual Simplex Algorithm
  • 113.  The (primal) simplex algorithm starts feasible but nonoptimal. The dual simplex starts better than optimal but infeasible. What if an LP model starts both nonoptimal and infeasible? Of course we can use artificial variables and artificial constraints to secure a starting solution. This is not mandatory, however, because the key idea of both the primal and dual simplex methods is that the optimum feasible solution, when finite, always occurs at a corner point (or a basic solution). 113 3.3.2 – Generalized Simplex Algorithm
  • 114.  This suggests that a new simplex algorithm can be developed based on tandem use of the dual simplex and primal simplex methods. First, use the dual algorithm to get rid of infeasibility (without worrying about optimality). Once feasibility is restored, the primal simplex can be used to find the optimum. Alternatively, we can first apply the primal simplex to secure optimality (without worrying about feasibility) and then use the dual simplex to seek feasibility. 114 3.3.2 – Generalized Simplex Algorithm
  • 115. Exercise 3.8  Solve the following problem by using the generalized simplex algorithm. Maximize z = 2x3 subject to -x1 + 2x2 - 2x3 ≥ 8 - x1 + x2 + x3 ≤ 4 2x1 - x2 + 4x3 ≤ 10 x1 , x2 , x3 ≥ 0 115 3.3.2 – Generalized Simplex Algorithm
  • 117.  In the sensitivity analysis section, we dealt with the sensitivity of the optimum solution by determining the ranges for the different parameters that would keep the optimum basic solution unchanged.  In this section, we deal with making changes in the parameters of the model and finding the new optimum solution. 117 3.4 – Post-Optimal Analysis
  • 118.  The following table lists the cases that can arise in post-optimal analysis and the actions needed to obtain the new solution (assuming one exists): 118 3.4 – Post-Optimal Analysis Condition after parameters change Recommended action Current solution remains optimal and feasible. No further action is necessary. Current solution becomes infeasible. Use dual simplex to recover feasibility. Current solution becomes nonoptimal. Use primal simplex to recover optimality. Current solution becomes both nonoptimal and infeasible. Use the generalized simplex method to obtain new solution.
  • 119.  The TOYCO model of Example 3.3 will be used to explain the different procedures. Recall that the TOYCO model deals with the assembly of three types of. toys: trains, trucks, and cars. Three operations are involved in the assembly. 119 3.4 – Post-Optimal Analysis TOYCO primal TOYCO dual Max 𝑧 = 3𝑥1 + 2𝑥2 + 5𝑥3 Min 𝑤 = 430𝑦1 + 460𝑦2 + 420𝑦3 subject to subject to 𝑥1 + 2𝑥2 + 𝑥3 ≤ 430 (Operation 1) 𝑦1 + 3𝑦2 + 𝑦3 ≥ 3 3𝑥1 + 2𝑥3 ≤ 460 (Operation 2) 2𝑦1 + 4𝑦3 ≥ 2 𝑥1 + 4𝑥2 ≤ 420 (Operation 3) 𝑦1 + 2𝑦2 ≥ 5 𝑥1, 𝑥2, 𝑥3 ≥ 0 𝑦1, 𝑦2, 𝑦3 ≥ 0 Optimum solution: Optimum solution: 𝑥1 = 0, 𝑥2 = 100, 𝑥3 = 230, 𝑧 = $1350 𝑦1 = 1, 𝑦2 = 2, 𝑦3 = 0, 𝑤 = $1350
  • 120.  The associated optimum tableau for the primal is given as 120 3.4 – Post-Optimal Analysis Basic X1 X2 X3 X4 X5 X6 Solution z 4 0 0 1 2 0 1350 X2 -1/4 1 0 ½ -1/4 0 100 X3 3/2 0 1 0 ½ 0 230 X6 2 0 0 -2 1 1 20
  • 121.  The feasibility of the current optimum solution may be affected only if (1) the right-hand side of the constraints is changed, or (2) a new constraint is added to the model.  In both cases, infeasibility occurs when at least one element of the right-hand side of the optimal tableau becomes negative-that is, one or more of the current basic variables become negative. 121 3.4.1 – Changes Affecting Feasibility
  • 122. Changes in the right-hand side.  This change requires recomputing the right-hand side of the tableau using Formula 1 in previous section:  Recall that the right-hand side of the tableau gives the values of the basic variables. 122 3.4.1 – Changes Affecting Feasibility New RHS of tableau in iteration 𝑖 = Inverse in iteration 𝑖 × New RHS of constraint
  • 123. Example 3.11  Situation 1. Suppose that TOYCO is increasing the daily capacity of operations 1, 2, and 3 to 600, 640, and 590 minutes, respectively. How would this change affect the total revenue?  Situation 2. Although the new solution is appealing from the standpoint of increased revenue, TOYCO recognizes that its implementation may take time. Another proposal was thus made to shift the slack capacity of operation 3 (x6 = 20 minutes) to the capacity of operation 1. How would this change impact the optimum solution? 123 3.4.1 – Changes Affecting Feasibility
  • 124. Solution: Situation 1.  With these increases, the only change that will take place in the optimum tableau is the right-hand side of the constraints (and the optimum objective value). Thus, the new basic solution is computed as follows:  Thus, the current basic variables, x2, x3, and x6, remain feasible at the new values 140, 320, and 30 units. The associated optimum revenue is $1880. 124 3.4.1 – Changes Affecting Feasibility 𝑥2 𝑥3 𝑥6 = 1/2 −1/4 0 0 1/2 0 −2 1 1 600 640 590 = 140 320 30
  • 125. Situation 2.  The capacity mix of the three operations changes to 450,460, and 400 minutes, respectively. The resulting solution is:  The resulting solution is infeasible because x6 = -40, which requires applying the dual simplex method to recover feasibility. 125 3.4.1 – Changes Affecting Feasibility 𝑥2 𝑥3 𝑥6 = 1/2 −1/4 0 0 1/2 0 −2 1 1 450 460 400 = 110 230 −40
  • 126.  First, we modify the right-hand side of the tableau as shown by the shaded column. Notice that the associated value of z = 3 × 0 + 2 × 110 + 5 × 230 = $1370. 126 3.4.1 – Changes Affecting Feasibility Basic X1 X2 X3 X4 X5 X6 Solution z 4 0 0 1 2 0 1370 X2 -1/4 1 0 ½ -1/4 0 110 X3 3/2 0 1 0 ½ 0 230 X6 2 0 0 -2 1 1 -40
  • 127.  From the dual simplex, x6 leaves and x4 enters, which yields the following optimal feasible tableau. 127 3.4.1 – Changes Affecting Feasibility Basic X1 X2 X3 X4 X5 X6 Solution z 5 0 0 0 5/2 1/2 1350 X2 1/4 1 0 0 0 1/4 100 X3 3/2 0 1 0 1/2 0 230 X4 -1 0 0 1 -1/2 -1/2 20
  • 128.  The optimum solution (in terms of x1 , x2 , and x3) remains the same as in the original model. This means that the proposed shift in capacity allocation is not advantageous because it simply shifts the surplus capacity in operation 3 to a surplus capacity in operation 1. The conclusion is that operation 2 is the bottleneck and it may be advantageous to shift the surplus to operation 2 instead. The selection of operation 2 over operation 1 is also reinforced by the fact that the dual price for operation 2 ($2/min) is higher than that for operation 1 ( $l/min). 128 3.4.1 – Changes Affecting Feasibility
  • 129.  This section considers two particular situations that could affect the optimality of the current solution: 1. Changes in the original objective coefficients. 2. Addition of a new economic activity (variable) to the model. 129 3.4.2 – Changes Affecting Optimality
  • 130. Changes in the Objective Function Coefficients.  These changes affect only the optimality of the solution and requires recomputing the z-row coefficients according to the following procedure: 1. Compute the dual values using Method 2 in previous section. 2. Use the new dual values in Formula 2, to determine the new z-row coefficients. 130 3.4.2 – Changes Affecting Optimality
  • 131.  Two cases will result: 1. New z-row satisfies the optimality condition. The solution remains unchanged (the optimum objective value may change, however). 2. The optimality condition is not satisfied. Apply the primal simplex method to recover optimality. 131 3.4.2 – Changes Affecting Optimality
  • 132. Example 3.13  Situation 1. In the TOYCO model, suppose that the company has a new pricing policy to meet the competition. The unit revenues under the new policy are $2, $3, and $4 for train, truck, and car toys, respectively. How is the optimal solution affected? 132 3.4.2 – Changes Affecting Optimality
  • 133. Solution:  The new objective function is Maximize z = 2x1 + 3x2 + 4x3 Thus, (New objective coefficients of x2, x3, and x6) = (3, 4, 0) Using Method 2, the dual variables are computed as 133 3.4.2 – Changes Affecting Optimality (𝑦1 𝑦2 𝑦3) = (3 4 0) 1/2 −1/4 0 0 1/2 0 −2 1 1 = (3/2 5/4 0)
  • 134.  The z-row coefficients are determined as the difference between the left- and right-hand sides of the dual constraints (Fonnula 2). It is not necessary to recompute the objective-row coefficients of the basic variables x2, x3, and x6 because they always equal zero regardless of any changes made in the objective coefficients. 134 3.4.2 – Changes Affecting Optimality Obj. coeff. of 𝑥1 = 𝑦1 + 3𝑦2 + 𝑦3 − 𝟐 = 13/4 Obj. coeff. of 𝑥4 = 𝑦1 − 0 = 3/2 Obj. coeff. of 𝑥5 = 𝑦2 − 0 = 5/4
  • 135.  Note that the right-hand side of the first dual constraint is 2, the new coefficient in the modified objective function.  The computations show that the current solution, x1 = 0 train, x2 = 100 trucks, and x3 = 230 cars, remains optimal. The corresponding new revenue is computed as 2 × 0 + 3 × 100 + 4 × 230 = $1220. The new pricing policy is not advantageous because it leads to lower revenue. 135 3.4.2 – Changes Affecting Optimality
  • 136.  Situation 2. Suppose now that the TOYCO objective function is changed to Maximize z = 6x1 + 3x2 + 4x3  We have 136 3.4.2 – Changes Affecting Optimality (𝑦1 𝑦2 𝑦3) = (3 4 0) 1/2 −1/4 0 0 1/2 0 −2 1 1 = (3/2 5/4 0) Obj. coeff. of 𝑥1 = 𝑦1 + 3𝑦2 + 𝑦3 − 𝟔 = −3/4 Obj. coeff. of 𝑥4 = 𝑦1 − 0 = 3/2 Obj. coeff. of 𝑥5 = 𝑦2 − 0 = 5/4
  • 137.  The new reduced cost of x1 shows that the current solution is not optimum. To determine the new solution, the z-row is changed as highlighted in the following tableau: 137 3.4.2 – Changes Affecting Optimality Basic X1 X2 X3 X4 X5 X6 Solution z -3/4 0 0 3/2 5/4 0 1220 X2 -1/4 1 0 ½ -1/4 0 100 X3 3/2 0 1 0 ½ 0 230 X6 2 0 0 -2 1 1 20
  • 138.  The highlighted elements are the new objective coefficients and new objective value. All the remaining elements are the same as in the original optimal tableau. The new optimum solution is then determined by letting x1 enter and x6 leave, which yields x1 = 10, x2 = 102.5, x3 = 215, and z = $1227.50.  Although the new solution recommends the production of all three toys, the optimum revenue is less than when two toys only are manufactured. 138 3.4.2 – Changes Affecting Optimality
  • 139. Exercise 3.9:  Consider the following LP:  a) Compute the entire simplex tableau associated with 139 . 0 , , 3 2 1 . . 2 max 3 2 1 2 1 3 2 3 1 3 2 1           x x x x x x x x x t s x x x z Basic variables = (𝑥1 , 𝑥2 , 𝑥3) and Inverse = 1 2 − 1 2 1 2 − 1 2 1 2 1 2 1 2 1 2 − 1 2
  • 140.  b) check for feasibility and optimality of solution from part (a).  c) Find the range of values of c1 (the contribution to profit of x1) for which the current basis remains optimal, assuming that all other parameters remain unchanged.  d) Find the range of values of b3 (the right hand side of constraint 1) for which the current basis remains optimal, assuming that all other parameters remain unchanged. 140

Editor's Notes

  1. Ro takhte vase d1 va d3